Sunteți pe pagina 1din 116

Pharmacology module-

1
Zaporozhye State Medical University , Ukraine, Edited by Rag@Puja 3rd course
2009-10
Grreddy836@gmail.com grreddy836@yahoo.com

1. Indicate the type of action of a drug, which is introduced orally,


is absorbed into the blood in intestine and acts on the certain organs
or tissues:
A. General (Resorptive)
B. Reflex
C. Main
D. Local (Topical)
E. Indirect
2. The formation of a secondary mediator is obligatory in membrane-
intracellular mechanism of hormone action. Point out the substance
that is unable to be a secondary mediator:
A. Diacylglycerol
B. Glycerol
C. CAMP
D. Ca2+
E. Inositol-3.4.5-triphosphate

3. What from the specified parameters is an obligatory condition of


fast permeation of the medicinal agents through a blood-brain barrier?
A. Short half-life
B. Water solubility
C. Lipid-solubility
D. Strong binding to proteins
E. Ionized state

4. Which of the drugs readily penetrate into the CNS?


A. Ionized drugs
B. Polar drugs
C. Lipid-soluble
D. Binding drugs to plasma proteins
E. Hydrophilic drugs with a non-uniform distribution of electrons

5. The patient of 37 years, suffering an obliterating endarteritis of


the legs, receives phenylin in a dose of 60 mg/kg. Because of
convulsion development (brain trauma is in anamnesis), phenobarbital
had been given, after the cancellation of which the nasal bleeding
occurred at the patient. The given complication is connected with:
A. Conjugation of phenylin with a glucuronic acid
B. Inhibition by phenobarbital of enzymes microsomal oxidations in a
liver
C. Induction by phenobarbital of enzymes microsomal
oxidations in a liver
D. Oxidative deamination of phenylin
E. Aliphatic hydroxylation of phenobarbital

6. Pharmacokinetics includes all of the following processes EXCEPT:


A. Absorption
B. Distribution
C. Biotransformation
D. Elimination
E. Clinical use

7. How pharmacological activity of a drug with high affinity to plasma


protein will be altered at hypoalbuminemia?
A. Will slightly decrease
B. Will increase
C. Will greatly decrease
D. Will not change
E. Will disappear

8. The bioavailability is defined as an average level if one is equal:


A. 0-20%
B. 40-70%
C. 100%
D. > 70%
E. < 40%

9. For prevention remote relapses of 4-days malaria a 42 y.o. patient


was given primaquine. On the 3-d day of the treatment with therapeutic
doses of the drug patient experienced abdominal pain, cardiac pain,
dyspepsia, generalized cyanosis. What is the reason of these adverse
effects of the drug?
A Cumulation of the therapeutic agent
B. Slowing down of drug‘s secretion with urine
C. Decreasing of activity of liver microsomal enzymes
D. Genetic deficiency of Glucose-6-phosphate
dehydrogenase
E. Potentiation of the drug‘s action by other therapeutic agent

10. Which one of the following is TRUE for a drug whose elimination
from plasma shows zero-order kinetics?
A. The half-life of the drug is proportional to the drug concentration
in plasma
B. The amount eliminated per unit time is constant
C. The rate of elimination does not depend on the
plasma concentration
D. Elimination involves a rate-limiting enzymatic reaction operating
at its maximal velocity
E. A plot of drug concentration versus time is a straight line

11. Choose the correct statement:


A. Weak bases are absorbed efficiently across the epithelial cells of
the stomach
B. Coadministration of atropine speeds the absorption of a second drug
C. Drugs showing large volume of distribution can be efficiently
removed by dialysis of the plasma
D. Stress emotions can lead to a slowing of drug
absorption
E. If the volume of distribution for a drug is small, most of the drug
is in the extraplasmic space

12. A drug, given in a 100 mg single dose, results in a peak plasma


protein concentration of 20 micro gram/ml.The apparent vol. of
distribution is:
A. 0.5 L
B. 1 L
C. 2 L
D.5L
E. 10 L

13. What is the main mechanism of drug absorption in the GIT?


A. Filtration
B. Passive diffusion
C. Active transport
D. Pinocytosis
E. All above

14. What reactions may occur only in repeated introductions of drugs?


A. Potentiation
B. Cumulation
C. Tolerance
D. Tachyphylaxis
E. All above except A
15. How is the drug action, which causes congenital malformations,
called?
A. Mutagenous action
B. Embriotoxic action
C. Teratogenic
D. Fetotoxic
E. All above

16. How to explain the reduction of effects of salicylates in the


interaction with Phenobarbital?
A. Inhibition of enzymes of the liver
B. Antagonist effect
C. Tachyphylaxis
D. Tolerance
E. Activation of enzymes of the liver
17. The breadth of therapeutic action is a range of doses:
A. From average therapeutic dose to minimum toxic dose
B. From single dose to course dose
C. From minimum effective (threshold) dose to minimum toxic dose
D. From single dose to daily dose
E. From minimum effective dose to average toxic dose

18. Give the correct explanation of the term ―tolerance‖:


A. Strong wish of taking the drug
B. Amplification of drug action in repeated introduction
C. Lowering of drug action in repeated introduction
D. Increased sensibility to the drug
E. All above

19 What is the acetylation of drugs?


A. Conjugation with glucuronic acid
B. Oxidation with participation of cytochrome P-450
C. Binding to acetic acid
D. Hydrolysis of drugs
E. Binding to glucuronic acid

20. What is the glucuronidation of drugs?


A. Conjugation with glucuronic acid
B. Oxidation with participation of cytochrome P-450
C. Binding to acetic acid
D. Hydrolysis of drugs
E. Cyclization of drugs

21. What does the term «tachyphylaxis» mean?


A. Strong wish of taking the drug
B. Amplification of drug action in repeated introduction
C. Lowering of drug action in repeated introduction
D. Increased sensibility to the drug
E. Rapid decrease of the effect in repeated
introduction of the drug within short period of time
22. The cumulation of drugs is possible:
A. In decreased reabsorption in kidney
B. In increased secretion in renal tubules
C. In lowering of the glomerular filtration
D. In activation of microsomal enzymes of the liver
E. In increase of biotransformation

23. A patient took PO half of the glass of chlorophos (strong


inhibitor of the acetylcholinesterase) with the aim of suicide.
Besides of gastric lavage, cleansing enema, infusion therapy, the
injection of atropine was made. As a result of the signs of the
intoxication completely disappeared.
Determine the type of interaction between atropine and chlorophos:
A. Inhibition of enzymes of the liver
B. Antagonist effect
C. Tachyphylaxis
D. Tolerance
E. Activation of enzymes of the liver

24. A patient took the long-lasting course of antibiotic therapy in


connection with infectious disease. During treatment he began to
complain of skin rash, itching, rhinitis. After the next injection of
the antibiotic asphyxia, edema of lips, increase of body to were
observed. The doctor abolished the antibiotic therapy and indicated
necessary drugs. Determine the type of this pathological condition:
A. Inhibition of enzymes of the liver
B. Sensibilization
C. Tachyphylaxis
D. Tolerance
E. Antagonist effect

25. Which of the following statements result in a doubling of the


steady-state concentration of a drug?
A. Doubling the rate of infusion
B. Maintaining the infusion rate, but doubling the loading dose
C. Doubling the rate of infusion and doubling the concentration of the
infused drug
D. Tripling the rate of infusion
E. Quadrupling the rate of infusion

26. Azithromycin, an antibiotic, has an apparent volume of


distribution (Vd) of approximately 30 L/kg. The correct interpretation
of this information is that Azithromycin is which of the following?
A. Effective only when given intravenously
B. Extensively distributed to sites outside the
vascular and interstitial spaces
C. Eliminated mainly by renal excretion, without prior metabolism
D. Unable to cross the blood-brain or placental barriers
E. Not extensively bound to plasma proteins

27. Which of the following administration routes most likely subject a


drug to a ―first-pass‖ effect in the liver?
A. Inhalation
B. Intramuscular
C. Intravenous
D. Oral
E. Sublingual

28. You are planning to infuse a drug IV at a constant amount per unit
time (rate). It has a first order elimination rate constant (kel) of
0.35/h. No loading dose will be given. Approximately how long will it
take for blood levels to reach steady state after the infusion begins?
A. 0.7 hours
B. 1.2 hours
C. 9 hours
D. 24 hours
E. 36 hours

29. Two drugs act on the same tissue or organ via activation of
different receptors, resulting in effects that are qualitatively the
opposite of one another. What is the type of drug antagonism?
A. Chemical
B. Physiologic
C. Dispositional
D. Pharmacologic
E. Competitive

30. Indicate chemical reaction of drug metabolism which belongs to the


phase of synthetic reactions (conjugation):
A. Deamination
B. Dealkylation
C. Oxidation
D. Glucuronidation
E. Reduction

31. What is the mechanism of absorption of a drug substance possessing


lipophilic properties?
A. Active transport
B. Passive diffusion
C. Filtration
D. Pinocytosis
E. Binding to transport proteins

32. Excretion of drugs or their metabolites across the cell membrane


occurs by the following mechanism only:
A. Passive diffusion
B. Active transport
C. Filtration
D. Pinocytosis
E. Simplified diffusion

33. If the drug substance or toxin inhibits synthesis of ATP, what


mechanism of penetration through the cell membrane is blocked?
A. Passive diffusion
B. Active transport
C. Filtration
D. Pinocytosis
E. Simplified diffusion

34. What pharmacokientic property is characteristic for lipophilic


drug substance?
A. It is metabolized in liver
B. Low bioavailability in oral introduction
C. Quick renal elimination
D. Low permeability across tissue barriers
E. Low renal reabsorption
35. Pharmacological incompatibility of medicines, which is found out
at a level of specific effect (for example, administration of
adrenomimetics and adreno blockers), is known as:
A. Physical
B. Pharmacokinetic
C. Chemical
D. Pharmacodynamic
E. Pharmaceutical

36. Influence of certain unfavorable factors, in particular some


medical agents, which precede pregnancy, enlarge risk of a birth of
the child with genetic defects. How this action is called?
A. Embriotoxic effect
B. Mutagenic effect
C. Teratogenic effect
D. Fetotoxic effect
E. Blastomogenic effect

37. It is known that in certain people with genetically determined


insufficiency of glucose-6-phosphate dehydrogenase the administration
of certain antimalarial agents can cause hemolysis. What is the name
of this atypical reaction?
A. Idiosyncrasy
B. Sensibilization
C. Allergic reaction
D. Tachyphylaxis
E. Tolerance

38. In a patient‘s liver the processes of detoxification of natural


metabolites and xenobiotics is broken. Activity of what cytochrome can
be reduced?
A. Cytochrome B
B. Cytochrome B -1
C. Hemoglobin
D. Cytochrome oxidase
E. Cytochrome P-450
39. Morphine undergoes glucuronidation in the liver. Binding of the
medicinal preparation with glucuronic acid during its
biotransformation will:
A. Increase its pharmacodynamic activity
B. Increase its water solubility
C. Reduce its water solubility
D. Not change its pharmacodynamic activity
E.

40. The patient was given a drug ―A‖. After a few days the effect of
the drug singnificanty dropped and for restoring of the initial effect
it was necessary to increase the drug‘s dose. What is the name of this
phenomenon?
A. Cumulation
B. Tachyphylaxis
C. Dependence
D. Tolerance
E. Idiosyncrasy

41. At embryonal period metabolism of the medicines is slower that in


adults. This peculiarity of fetus pharmacokinetics linked with:
A. Functional prematurity of majority of enzymes or
their absence
B. Higher permeability of histohematologic barriers
C. Significant volume of extracellular fluid
D. Ability of the skin to absorb and excrete water-soluble medicines
E. Development of organs receptors at different terms

42. A patient who was been suffering from cardiac insufficiency for
several months has been taking digoxin on an outpatient basis. At a
certain stage of treatment there appeared symptoms of drug overdose.
What phenomenon underlies the development of this complication?
A. Tachyphylaxis
B. Sensibilization
C. Functional cumulation
D. Habituation
E. Material cumulation
43. A 36 y.o. man has a craniocerebral trauma. Objectively: diminished
breath sounds, thready pulse, and no reflexes. What way of pyracetam
introduction will be the most appropriate in this case?
A. Inhalation
B. Subcutaneous
C. Rectal
D. Intravenous
E. Peroral

44. A patient, who has been treating with Clophelinum (Clonidine) due
to arterial hypertension, has taken an alcoholic drink that has caused
the sharp inhibition of the CNS. What is the name of this interaction?
A. Potentiation
B. Summation
C. Cumulation
D. Intoxication
E. Idiosyncratic reaction

45. A patient who has been treated with diazepam on account of


neurosis complains of toothache. Doctor administered him an analgetic,
but its dose was lower than average therapeutic dose. What phenomenon
did the doctor take into account while prescribing the patient an
underdose?
A. Tolerance
B. Cumulation
C. Summation
D. Potentiation
E. Drug dependence

46. During surgical operation with administration of a myorelaxant


(neuromuscular blocker), a disturbance of breathing developed.
Administration of proserine (neostigmine) has improved patient‘s
condition. What is the name of this type of drug interaction?
A. Incompatibility
B. Antagonism
C. Tachyphylaxis
D. Synergism
E Cumulation

47. Doctor prescribed 5% ephedrine nasal drops to the patient


suffering from acute rhinitis. After the first introduction of the
drops nasal breathing of the patient was improved. He started to put
the drops every 30 min. As a result there was sharp weakening of the
effect of the preparation. Doctor stated appearance of tachyphylaxis
to often introduction of the drops. Indicate the reason of appearance
of tachyphylaxis:
A. Exhaustion of substrate through which ephedrine acts
B. Adaptation of cell receptors
C. Increase of activity of hepatic enzymes
D. Increase of excretion of ephedrine
E. Blockade of receptors

48. Indicate the number of introduction of a drug per day if its half-
life period (T1/2) is 20 hours:
A. 2 times a day
B. 3 times a day
C. 4 times a day
D. Once daily
E. Every 12 hours

49. What is half-life period (T1/2) of a drug?


A. Blood plasma volume, which is cleared from the drug in a period of
time
B. Period during which concentration of a drug in blood
plasma is decreased by 50%
C. Period of complete excretion of a drug from the organism
D. Speed of excretion of the drug via kidneys
E. Ratio between speed of excretion and blood plasma concentration of
a drug

50. Benzylpenicilline natrii salt is injected to the patient with


pneumonia in a doze 500,000 UA 6 times per day. Just after an
injection the patient developed fever, spasms, loss of consciousness.
What has happened to the patient?
A Anaphylactic shock
B. Idiosyncrasy
C. Tachyphylaxis
D. Cumulation
E. Tolerance

51. The patient suffering from epilepsy had been receiving daily 0,2 g
of phenobarbitale continuously. Recently attacks became more frequent,
suppressed mood is observed. Which process became the cause of
deterioration of the patient‘s state?
A. Liver monooxygenase enzymes system induction.
B. Liver monooxygenase enzymes system inhibition.
C. Lipolysis activation.
D. Gluconeogenesis activation.
E. Glycolysis inhibition.

52. Ambulance has been called to a 22 y.o. man, because of bronchial


asthma attack. What way of salbutamole administration is the most
appropriate in this case?
A. Intravenous
B. Intramuscular
C. Inhalation
D. Subcutaneous
E. Sublingual

53. A stenocardia attack happened with a 48 y.o. man. The doctor has
prescribed him a pill of nitroglycerine sublingually. Why has the
doctor chosen a sublingual method of the medicine administration?
A Bad intestinal absorption
B Drug elimination by gastric juice
C Effect develops faster
D. It is activated by saliva
E. Only because it damages the liver

54. Phenobarbital was prescribed to a 54 y.o. man, suffering from


insomnia. The sleep rhythm has been normalized. But gradually, during
2 months, the effect of a medicine has decreased and insomnia
reoccured. What factor causes the reduction of somnifacient medication
effect?
A.Bad solubility
B.Tolerance
C.Bad absorption in the stomach
D.Accumulation in lipids
E.Tachyphylaxis

55. During the embryonic period metabolism of medications happens


considerably slower than in an adult organism. The specified feature
of an embryo pharmacokinetics is caused, first of all, by the
following:
A.The functional imperfection of enzymes, predominantly by their
absence
B The big permeability of histohematologic barriers
C Essential volume of extracellular liquid
D. Ability for absorbtion and allocation of water-soluble drugs by the
skin
E. „Maturing‖ of receptors in organs in different terms

56. It is known,that in people with genetically caused insufficiency


of glucose-6-phosphate dehydrogenase enzyme of red blood cells, lysis
of stated cells may develop as a reaction to some antimalaric
medications injection. How is such untypical reaction to medications
designated?
A.Sensibilisation
B.Idiosyncrasy
C.Allergic reaction
D.Tachyphylaxis
E.Tolerance

57. How can the fact that for tuberculosis treatment the isoniazide
doze is selected individually with the obligatory control of
concentration of the medication in urine after its first application
be explained?
A.Development of haemolytic anaemias
B.Hyperglycemia after drug administration
C.Development of renal insufficiency
D.Irritating effect of the drug
E.Genetically caused methylation of the drug

58. The patient with hypertension has been taking an antihypertensive


medications for a long time, but suddenly he has stopped doing it.
After that patient‘s condition has worsened, hypertensive crisis has
developed. What kind of a collateral action is it?
A Cumulation
B Syndrome of a cancellation
C Tolerance
D Sensibilisation
E Addiction

59. During the visit of a stomatologist, an attack of bronchospasm has


developed in the patient who has been treated by 5 % efedrine
hydrochloride solution injection. In 20 min the attack reoccured. The
additional injection of efedrine had no effect due to tachyphylaxis.
What mechanism underlies this phenomenon?
A Exhaustion of the noradrenaline depot in presynaptic endings
B Inhibiton of adrenal receptors
C Activation of adrenal receptors
D Material cumulation of the drug
E Induction of microsomal enzyme system by the liver

60. A long course of treatment with cyancobalamine was administered IM


to the patient with megaloblastic anemia developed after gastroectomy.
What advantage does a parenteral way of administration have over oral?
A The drug is quickly deduced
B The drug is quickly absorbed
C The drug circulates in blood for a longer time
D The drug does not disintegrate in the liver
E.This way is effective in absence of gastromucoprotein

61. Which of the following conditions is NOT characteristic of cocaine


over dosage?
A. Dilation of the pupil
B. Euphoria
C. Tachycardia
D. Peripheral vasodilation
E. Hallucinations

62. A patient was delivered to the surgical department in connection


with acute appendicitis. He was proposed to make appendectomy under
the local anesthesia. But from his medical history was known that he
had severe allergic reactions to drugs. Choose the drug the most
preferable for the infiltration anesthesia in this situation:
A. Novocaine
B. Lidocaine
C. Anesthesine
D. Dicaine
E. Any from indicated above

63. A patient was delivered to the ophthalmologic department with


trauma of the eye as a result of hitting with metallic shaving. Edema
of the eye, significant hyperemia, hyperlacrimation, pain were
observed. Choose the local anesthetic to use topically for removal of
the foreign body:
A. Procaine (Novocaine)
B. Trimecaine
C. Anesthesine
D. Dicaine
E. Ultracaine

64. A 23-year-old woman with red and itchy eczematous dermatitis


visits a doctor office. She had a dental procedure one day earlier
with administration of a local anesthetic. There were no other
findings, although she indicated that she had a history of allergic
reactions. Which of the following drugs is most likely involved?
A. Lidocaine
B. Ultracaine
C. Bupivacaine
D. Novocaine (Procaine)
E. Trimecaine

65. A patient with renal failure has a periodic hemodialysis while


awaiting a transplant. Between dialysis sessions we want to reduce the
body‘s phosphate load by reducing dietary phosphate absorption and
removing some phosphate already in the blood. Which compound shall we
administer orally?
A. Aluminum hydroxide
B. Magnesium hydroxide
C. Sodium bicarbonate
D. Tannin
E. Bismuth subnitras

66. Bismuth salts are thought to be effective adjuncts in managing, if


not healing, refractory gastric ulcer because they have bactericidal
properties against:
A. Staphylococcus aureus
B. Clostridium difficile
C. E.coli
D. Helicobacter pylori
E. Bactericides fragilis

67. Indicate the mechanism of action of local anesthetics:


A. Formation of albuminates with tissue‘s proteins
B. Blockade of Na+-channels
C. Blockade of M-cholinoceptors
D. Inhibition of non-specific excitatory systems of CNS
E. Blockade of (-adrenoceptors

68. As a result of the influence of terminal anesthesia which part of


the skin and mucus membranes are affected:
A. Sensory nerve endings
B. Epidermis
C. Subcutaneous fatty tissue
D. Walls of capillaries
E. Derma

69. Indicate the principle of action of covering drugs:


A. Formation of protective layer on the mucous membranes
B. Blockade of mucous membranes receptors
C. Coagulation of proteins of superficial layer of mucous membrane
D. Formation of complexes with toxic agents
E. Stimulation of regenerative processes

70. For anesthesia during tooth extraction solution of


novocaine(procaine) has been used. Why it was not introduced into
gingival tissue, but in the projection of nerve fiber path? Because
of:
A. Transmission of pain impulses is blocked
B. Alteration of tissue pH in anesthesia area
C. Depression of axonal transport
D. Disturbance of action potential formation of the pain receptors
E. Increasing of excitability of the pain receptors

71. Why novocaine is not used for terminal anaesthesia?


A Is poorly absorbed through normal skin surface and mucous
membrane
B. Doesn‘t cause covering action
C It is fast absorbed and inhibits the CNS
D. Irritates mucous membrane
E. Activates M-cholinoceptors

72. Indicate main effect of the local anesthetics:


A. Eliminate all kinds of sensibility due to blockade of action
potential
B. Selective relieve of pain sensibility in local action
C. Decrease of excitability of nerve endings
D. Decrease of excitability and conductivity of the afferent NS
E. Eliminate all kinds of sensibility due to paralysis of the CNS

73. Indicate the main effect of astringent drugs.


A. Decrease of nerve ending sensibility
B. Reduction of hyperemia of mucous membranes due to vasoconstriction
C. Relief of pain
D. Decrease of synthesis of histamine
E. Decrease of gland secretion

74. What drugs from the group of local anaesthetics are not used
together with sulfonamides?
A. Novocaine (procaine)
B. Sovcainum
C. Lidocaine
D. Trimecaine
E. Ultracaine

75. Determine the drug, which is used for all types of anaesthesia.
A. Novocain (procaine)
B. Anesthesine (benzocaine)
C. Lidocaine
D. Trimecaine
E. Dicaine (tetracaine)

76. A patient with increased sensitivity to sulfonamide needs tooth


extraction. What local anesthetic has to be used?
A. Anesthesine (benzocaine)
B. Dicaine (tetracaine)
C. Lidocaine
D. Cocaine
E. Novocainum (procaine)

77. Injection of a local anesthetic has to be given to a patient for


tooth extraction. What drug from listed below is to be chosen?
A. Cocaine
B. Dicaine (tetracaine)
C. Anesthesine (benzocaine)
D. Lidocaine
E. Ketamine

78. This agent is poorly soluble in water, so it is used for


superficial anesthesia only in the form of ointment, paste and powder.
What is this drug?
A. Sovcainum
B. Novocainum (procaine)
C. Pyromecaine
D. Trimecaine
E. Anesthesine (benzocaine)

79. What drug has to be added to lidocaine solution to prolong its


action?
A. Adrenaline
B. Caffeine
C. Analginum (methamizole)
D. Atropine
E. Anaprilinum (Propranolol)

80. What is the main indication for adsorbing drugs use?


A. Diarrhea
B. Hypoacidic gastritis
C. Decrease in trypsin activity
D. Decrease in bile secretion
E. Intoxication

81. A nurse used mustard plaster with water of more than 600C to and
applied it on patient‘s back. In 30 min she found that patient‘s skin
under the mustard plaster did not get red. What is the reason for
absence of mustard plaster effect?
A. Inactivation of mirosine
B. Inactivation of choline esterase
C. Activation of mirosine
D. Inactivation of monoaminooxydase
E. Activation of methyltranspherase

82. It is necessary to carry out local anesthesia by lidocaine at


extraction of a foreign body from an eye. What is the action mechanism
of this drug?
A. Blocks Na+ channels
B. Breaks transit of K+ through the membrane
C. Decreases dehydrogenases activity
D. Activate K+ channels
E. Activate Na+ channels
83. A farmer has been delivered to hospital with a sharp pain in his
eye. What local anesthetic will cause the strongest anesthesia at
terminal anesthesia?
A. Novocaine (Procaine)
B. Lidocaine
C. Trimecaine
D. Dicaine (Tetracaine)
E. Anaesthesine (Benzocaine)

84. Which of the following drugs incorrectly matches a cholinergic


agonist with a pharmacologic action?
A. Proserine: stimulates atonic bladder
B. Carbachol: induces release of epinephrine from the adrenal medulla
C. Acetylcholine: decreases heart rate and cardiac output
D. Pilocarpine: reduces intraocular pressure
E. Physostigmine: decreases intestinal motility

85. Which of the following signs is NOT an expected symptom of


poisoning with Neostigmine (Proserine)?
A. Increase in skeletal muscle tonus
B. Increased bronchial secretions
C. Miosis
D. Tachycardia
E. Convulsions

86. Select the most appropriate drug that depolarizes neuromuscular


end plate:
A. Dithylinum
B. Diplacin
C. Tubocurarine
D. Mellictin
E. Pipecuronium (Arduan)

87. Select the most appropriate drug that reverses the effects of
nondepolarising blockers:
A. Dithylinum
B. Proserine (Neostigmine)
C Fresh blood transfusion
D Scopolamine
E. Ipratropium bromide

88. Select the most appropriate drug to treat the poisoning with
depolarising blockers:
A. Dithylinum
B. Neostigmine
C Fresh blood transfusion
D Scopolamine
E. Ipratropium bromide

89. Which ONE of the following drugs would be useful in the long-term
treatment of myasthenia gravis?
A. Arduan
B. Atropine
C. Proserine (Neostigmine)
D. Scopolamine
E. Lobeline

90. A 40-year-old male farm worker is brought to the emergency room.


He was found confused in the orchard and since then has lost
consciousness. His heart rate is 45 and his BP is 80/40 mm Hg. He is
sweating and salivating profusely. Which of the following treatments
is indicated?
A. Galantamine
B. Norepinephrine
C. Proserine
D. Atropine sulfate
E. Ipratropium bromide

91. A patient of 20 years old was delivered to a hospital with sharp


pain in the abdomen, vomiting, impeded breathing, and dyspnoea. During
the patient‘s examination the following data were revealed: the skin
is moist, hypersalivation, miosis, bradycardia muscular twitching.
From the anamnesis it is known that the patient is a toxoman, he
breathed in the aerosol for extermination of insects. What mechanism
of action has the agent, which has caused poisoning?
A. M-cholinoblocking
B. N-cholinoblocking
C. M-cholinomimetic
D. N-cholinomimetic
E. M, N-cholinomimetic

92. The patient with complaints of constant dry mouth addressed a


dentist. The doctor made the diagnosis «xerostomia». To normalize the
secretion of saliva he administered Pilocarpine 5-6 drops 3 times a
day. But the patient violated the instructions and took the drug
without following the indicated dosage. Soon besides the disappearance
of mouth dryness he felt cramps, diarrhea, increased bronchial
secretion, sweating, tearing, bradycardia, constriction in the chest,
involuntary urination, hypotension. With the help of what drug can the
side effects be easily eliminated?
A. Aceclidine
B. Cytiton
C. Lobeline
D. Atropine
E. Proserine

93. The effects of tubocurarine can be antagonised by:


A. Pilocarpine
B. Aceclidine
C. Cytiton
D. Neostigmine (Proserine)
E. Lobeline

94. All of following side effects are seen with Ipratropium bromide
except:
A. Urinary retention
B. Dryness of mouth
C. Scratching in trachea
D. Miosis
E. Glaucoma

95. The sub-type of cardiac muscarinic receptor is predominantly:


A. M1
B. M2
C. M3
D. M1 and M3
E. M1, M2 and M3

96. Which of the following anticholinergic is claimed to act


selectively on bronchial muscle?
A. Isadrine
B. Ipratropium bromide
C. Salbutamole
D. Pirenzepine
E. Ephedrine

97. Physostigmine (Eserini sulfas) which is used mainly for its


parasympathomimetic effects, inhibits:
A. Tyrosine hydroxylase
B. Acetylcholinesterase
C. Catechol-O-methyltransferase (COMT)
D. Monoamine oxidase (MAO)
E. DOPA decarboxylase
98. A patient with renal colic has been administered a spasmolytic
agent from M-cholinoblockers. Specify this drug.
A. Platiphylline
B. Ephedrine
C. Pirenzepine
D. Galanthamine
E. No-spa

99. A patient receives Aceclidine after abdominal surgery. His heart


rate falls slightly and she experiences some wheezing. These pulmonary
and cardiac responses represent or reflect:
A. Expected side effect
B. Idiosyncrasy
C. Parasympathetic ganglion activation
D. Reflex (baroreceptor) suppression of cardiac rate
E. Undiagnosed asthma

100. We give an ‗effective dose‘ of atropine to a person who is


poisoned with an acetylcholinesterase inhibitor. Which structure will
continue to be overacted by the excess acetylcholine after this drug
is given?
A. Airway smooth muscle
B. S-A node of the heart
C. Salivary and lacrimal glands
D. Skeletal muscle
E. Vascular smooth muscle

101. Which is a muscarinic receptor-blocking drug that is administered


by inhalation to cause bronchodilation for patients with emphysema
[chronic obstructive pulmonary disease (COPD)]?
A. Platiphyllini hydrotartras
B. Ipratropium bromide
C. Pancuronium
D. Pilocarpine
E. Salbutamole

102. A patient with a history of asthma experiences significant


bronchoconstriction and urticaria, and histamine is a main mediator in
these responses. Which of the following drugs may pose extra risk for
this patient – not because it has any bronchoconstrictrictor effects
in its own right, but because it quite effectively releases histamine
from mast cells?
A. Atropine
B. Neostigmine sulfate
C. Propranolol
D. Pancuronium
E. d-Tubocurarine
103. A dentist prescribed an agent stimulating salivation to a patient
with xerostomia. Indicate the drug.
A. Pilocarpine
B.Dithylinum (suxamethonium)
C.Armine
D.Scopolamine
E.Atropine

104. A patient had to go through an operation. Doctors introduced him


dithylinum (listenone) and performed intubation. After the end of
operation and cessation of anesthesia the independent respiration
wasn‘t restored. Which enzyme deficit prolongs the action of muscle
relaxant?
A. Succinate dehadrogenase
B. K-Na-adenosine triphosphatase
C. Carbanhydrase
D. Pseudocholinesterase
E. N-acetyltransferase

105. A patient with fracture of his lower jaw was admitted to the
maxillofacial department. It was decided to fix his bones surgically
under anesthetic. After intravenous introduction of muscle relaxant
there arose short febrillar contractions of the patient‘s facial
muscles. What muscle relaxant was applied?
A. Diazepam
B. Tubocurarin chloride
C. Pipecuronium bromide
D. Dithylinum
E. Melictine

106. During surgical operation with administration of a myorelaxant


(neuromuscular blocker), a disturbance of breathing developed.
Administration of proserine (neostigmine) has improved patient‘s
condition. What is the name of this type of drug interaction?
A. Incompatibility
B. Antagonism
C. Tachyphylaxis
D. Synergism
E. Cumulation

107. Introduction of a pharmaceutical substance to an experimental


animal resulted in reduction of salivation, pupil mydriasis. Next
intravenous introduction of acetylcholine didn‘t lead to any
significant changes of heart rate. Name this substance:
A. Proserine
B. Adrenaline
C. Propranolol
D. Atropine
E. Salbutamol

108. The doctor has prescribed Cyclodolum for Parkinson disease


treatment. What is the mechanism of anti parkinsonian action of the
drug?
A. M cholinomimetic action
B. M cholinoblocker action
C. Stimulation of dopamine receptors
D. Blockade of dopamine receptors
E. (-Adrenoblocker action

109. The glaucoma is diagnosed in 43 y.o. patient for the first time.
For the treatment doctor has prescribed an anticholinesterase agent in
eye drops. Indicate this agent:
A. Galantamine
B. Physostigmine
C. Pilocarpine
D. Dipiroxim
E. Atropine

110. After use of neostigmine in the patient with myasthenia, such


symptoms as nausea, diarrhea, twitching of muscles of tongue and
skeletal muscles have appeared. What agent can abort the intoxication?
A. Physostigmine
B. Methacine
C. Mesatone (Phenylephrine)
D. Isadrine
E. Pyridostigmine bromide

111. M-cholinoblockers have quite wide application in modern clinical


practice. What case are M-cholinoblockers contraindicated at?
A. Renal colic
B. Atony of intestine
C. Bronchial asthma
D. Liver colic
E. Gastric ulcer

112. During reposition of the bones at a patient with forearm trauma


dithylin (succinylcholine) was used. Complete restoration of muscular
tonus and functions took more than an hour. What is the reason of the
significant prolongation of curare-like action of the agent?
A. Genetically determined deficit of butyrylcholinesterase
B. Yielding of an active metabolite
C. Genetically determined deficit of monoamine oxidase
D. Inhibition of microsomal oxidation
E. Genetically determined deficit of hydroxylase

113. During surgical operation a patient has developed symptoms of


dithylin (succinylcholine) overdosage. What treatment is necessary to
provide to the patient?
A. Administration of anticholinesterases
B. Administration of M-cholinoblockers
C. Administration of ganglioblockers
D. Blood transfusion
E. Administration of N-cholinoblockers

114. The patient with complaints on dryness in mouth, photophobia and


visual disturbances is delivered into an admission department. He has
also redness and dryness of skin, pupils‘ dilation and tachycardia. At
the further inspection, the following diagnosis has been established:
poisoning by Belladonna alkaloids. What agent from given medicines is
wise to use?
A. Armine
B. Pilocarpine
C. Proserine (Neostigmine)
D. Dipiroxim
E. Diazepam

115. A 63 y.o. patient has got an injection of pentamine for


hypertensive crisis. What is the action mechanism of the drug?
A. Stimulation of M-cholinoceptors
B. Inhibition of M-cholinoceptors
C. Inhibition of M-,N-cholinoceptors
D. Stimulation of N-cholinoceptors
E. Inhibition of N-cholinoceptors

116. At complex narcosis (general anesthesia) protocol during stomach


resection, a patient has got an injection of tubocurarine chloride as
a myorelaxant (neuromuscular blocker). What agent – antagonist should
be administered to the patient for restoration of his self-driven
respiration?
A. Proserine (Neostigmine)
B. Dithyline (Succinylcholine)
C. Atropine
D. Etimizol
E. Benzohexonium

117. Atropine is prescribed to the patient for the removing of


intestinal colic. What from the named diseases can be a
contraindication in this case? 2003
A. Hypotension
B. Glaucoma
C. Sinus bradycardia
D. Bronchial asthma
E. Giddiness (Dizziness)

118. A ganglioblocker - Pentamine has been administered to the patient


with hypertensive crises. What effects should doctor be worry about
after the drug‘s injection?
A. Suppression of respiration
B. Diarrhea
C. Rebound syndrome
D. Infringement of taste sensation
E. Orthostatic (postural) hypotension

119. A M1-cholinoblocker has been administered to the patient for the


treatment of gastric peptic ulcer. Indicate this agent:
A. Pirensepine (Gastrozepine)
B. Scopolamine
C. Ipratropium bromide
D. Methacine
E. Platiphylline

120. Curare-like myorelaxants (neuro-muscular blockers) are used in


clinical practice for decreasing and cessation of impulse transmission
from nerve ending to muscular fiber. What is the mechanism of action
of this group?
A. Blockade N-cholinoceptors of postsynaptic membrane of
neuromuscular junction
B. Inhibition of acetylcholinesterase
C. Inhibition of Na+/ K+ pump
D. Reduction of neurotransmitter release into synaptic cleft
E. Blockade of Ca2+ inward through presynaptic membrane

121. After utilization of ophthalmic drops patient with glaucoma has


miosis, myopia, and drop of intraocular pressure. Which pharmacologic
group can cause such effects?
A. M-cholinomimetics
B. M-cholinoblockers
C. N-cholinomimetics
D. Ganglioblockers
E. Adrenoblockers

122. The victim has been delivered to the traumatologic emergence


station by an ambulance with the diagnosis of closed fracture of an
average third of the hip with dislocation of bone fragments. To
reposit bone fragments, 10 ml of a 2% Ditiline solution was injected
IV whereupon a continuous apnea and myorelaxation has developed. What
enzyme deficiency causes such collateral action?
A. N-acetyltranspherase.
B. Uridinediphosphoglucuronosyl transferase.
C. Glucose-6-phosphate dehydrogenase.
D. Methemoglobine reductase
E. Pseudocholine esterase

123. A patient 50 y.o. with complaints of heart palpitation, pain in


the heart area, headache, vertigo was delivered to a hospital. During
the examination angina pectoris, disorders of cardiac rhythm
(paroxysmal tachycardia, atrium fibrillation) and arterial
hypertension (180/90 mm Hg) were revealed. Make the rational choice of
the drug in this situation.
A. Propranolol (Anapriline)
B. Epinephrine
C. Ephedrine
D. Neostigmine
E. Reserpine

124. A 60 y.o. asthmatic man comes in for a check-up and complains


that he is having some difficulty in «starting to urinate». Physical
examination indicates that the man has a BP of 160/100 mm Hg and a
slightly enlarged prostate. Which of the following medications would
be useful in treating both of these conditions?
A. Phentolamine
B. Reserpin
C. Ephedrine
D. Doxazosin
E. Propranolol

125. A doctor diagnosed hypoglycemic coma in a patient with diabetes


mellitus and administered glucose solution IV to him. Patient‘s
condition improved. What drug can be used additionally as a chemical
antagonist of insulin?
A. Dopamine
B. Mesatonum
C. Strophanthin
D. Adrenaline
E. Noradrenaline

126. Anaphylactic shock has delivered in a patient after novocaine


(procaine) injection. What agent suppresses histamine release from
mast cells and eliminates main symptoms of anaphylactic shock?
A. Ketotifen
B. Cromolin natrium
C. Adrenaline
D. Mesatonum
E. Noradrenaline

127. A 38-year-old female has taken unknown drug for decreasing high
BP. After its administration systolic pressure decreased but
tachycardia appeared. What was the patient administered?
A. Atenolol
B. Reserpine
C. Phentolamine
D. Propranolol
E.Prasosine

128. A patient has spasm of smooth muscles of the bronchi. Using of


what activators will be physiologically reasonable for emergency
treatment?
A. ±adrenoceptorS
B.D-receptors
C. N-cholinoceptorS
D.Beta adrenoreceptors
E.alfa n beta adrenoreceptors

_1_2_9_._ _A_ _2_4_-_y_e_a_r_-_o_l_d_ _p_a_t_i_e_n_t_ _h_a_s_


_b_r_o_n_c_h_i_a_l_ _a_s_t_h_m_a_._ _F_o_r_ _t_h_e_
_t_r_e_a_t_m_e_n_t_ _o_f_ _a_s_t_h_m_a_ _a_t_t_a_c_k_ _a_
_d_o_c_t_o_r_ _h_a_s_ _p_r_e_s_c_r_i_b_e_d_ _s_a_l_b_u_t_a_m_o_l_._
_W_h_a_t_ _i_s_ _t_h_e_ _m_e_c_h_a_n_i_s_m_ _o_f_ _c_u_r_a_t_i_v_e_
_a_c_t_i_o_n_ _o_f_ _t_h_e_ _a_g_e_n_t_?_
26A26.26 26±_-26 26a26n26d26 26²_-26r26e26c26e26p26t26o26r26s26
26s26t26i26m26u26l26a26t26i26o26n26
26B26.26 26²_226 26a26d26r26e26n26o26r26e26c26e26p26t26o26r26s26
26b26l26o26c26k26a26d26e26
26C26.26 26²_226
26a26d26r26e26n26o26c26e26p26t26o26r26s26
26s26t26i26m26u26l26a26t26i26o26n26
26D26.26 26²_126 26a26d26r26e26n26o26r26e26c26e26p26t26o26r26s26
26b26l26o26c26k26a26d26e26
26E26.26 26²_126 26a26d26r26e26n26o26r26e26c26e26p26t26o26r26s26
26b26l26o26c26k26a26d26e26

130. Alpha-1 receptor actions are mediated through:


A. AAMP
B. AGMP
C.Ca2+ions
D.ADP
E. K+ Channels

131. With excitation of which receptors are the following effects


associated dilation of blood vessels, decrease of tonicity of the
bronchial muscles and myometrium,stimulation of glycogenolysis?
A. ±1- adrenoceptors
B. ±2- adrenoceptors
C.Beta1
D.Beta 2
E.Beta3

132. An ambulance was called for a 22 y.o. patient with status


asthmatics. What route of adrenaline administration will be the most
appropriate in this case?
A. Intravenous
B. Inhalation
C. Subcutaneous
D. Sublingual
E. Intramuscular

133. All the following drugs are used topically in the treatment of
chronic wide-angle glaucoma. Which one reduces intraocular pressure by
decreasing the formation of the aqueous humor?
A. Timolol
B. Pilocarpine
C. Physostigmine
D. Neostigmine
E. Aceclidine

134. A 40 y.o. patient complaints of intensive heartbeats, sweating,


nausea, vision impairment, arm tremor, hypertension. From his
anamnesis: 2 years ago he was diagnosed with pheochromocytoma. What
hypotensive agent should be administered?
A. Captopril
B. Prazosine
C. Drotaverine
D. Nifedipine
E. Platophylline

135. A patient with II stage hypertension has been taking one of


hypotensive medications for the purpose of treatment. After a time AP
decreased, but the patient started complaining of flaccidity,
sleepiness, indifference. A bit later he felt stomach pain. He was
diagnosed with ulcer. What hypotensive medication has the patient been
taking?
A. Furosemide
B. Verapamil
C. Reserpine
D. Dibazole
E. Captopril

136. A patient ill with bronchial asthma didn‘t inform his doctor that
he had attacks of stenocardia. Doctor administered him a medication,
which taking resulted in less frequent attacks of bronchial asthma,
but stenocardia attacks became more frequent. What medication was
administered?
A. Salbutamol
B. Aminophylline
C. Isadrin
D. Cromolin sodium
E. Phenotherol

137. A patient with bronchial asthma has been given 0.5% solution of
isadrinum (isoproterenol). Bronchospasm was removed, but the patient
started to complain on a pain in cardiac area. It is linked to
stimulation of:
A. Acetylcholine synthesis
B. (1-adrenoceptors
C. beta 1- adrenoceptors
D. (2- adrenoceptors
E. M-cholinoceptors

138. The 42 y.o. patient has been suffering from bronchial asthma and
tachycardia for a long time. Administer the optimal drug for removing
of bronchospasm:
A. Adrenaline
B. Ephedrine
C. Orciprenaline
D. Isadrinum
E. Salbutamol

139. A patient has anaphylactic shock. Name a drug of choice.


A. Mesatone (Phenylephrene)
B. Adrenaline
C. Ephedrine
D. Noradrenaline
E. Fenoterol

140. The drug from group of beta-adrenoblockers has been prescribed to


47 y.o. patient for the treatment of ischemic heart disease. When will
beta-adrenoblockers be absolutely contraindicated?
A. Arterial hypertension
B. Bronchial asthma
C. Hypertrophic cardiomyopathy
D. Thyrotoxicosis
E. History of myocardial infarction

141. To the patient with an acute rhinitis 5% solution of ephedrine


has been prescribed. However, the patient instilled the drug in the
nose each 20-30 min. Because of development tachyphylaxis, effect of
the drug sharply decreased. What mechanism of underlies development of
this phenomenon?
A. Depletion of mediator depot in presynaptic endings
B. Activation of adrenoceptors
C. Blockade of adrenoceptors
D. Material cumulation
E. Induction of microsomal enzymes by liver

142. An ophthalmologist with diagnostic purpose (dilation of the pupil


for observation of eye bottom) has used 1% solution of mesatone
(phenylephrene). What mechanism determines mydriasis caused by the
agent?
A. Activation of (1-adrenoceptors
B. Activation of (2-adrenoceptors
C. Blockade of (1- adrenoceptors
D. Blockade of (1- adrenoceptors
E. Activation of M-cholinoceptors

143. A 64 y.o. woman with toxic goitre complains on permanent


palpitation. What agent should be administered for correction of the
heart rate?
A. Isadrine
B. Salbutamol
C. Pentamine
D. Propranolol (Anaprilinum)
E. Fenoterol

144. A patient has marked dropping of AP during surgical operation


with hygronium administration. Representatives of what pharmacological
group could normalize AP in given situation?
A. N-cholinomimetics
B beta 1 -adrenomimetics
C. (1- adrenoblockers
D. (2- adrenomimetics
E. M-cholinomimetics
145. After tooth extraction patient has developed local bleeding. A
dentist applied sponge with a medicine and bleeding subsided. What
medicine did the dentist use?
A. Isadrine
B. Salbutamol
C. Adrenaline
D. Prazosin
E. Timolol

146. A patient with chronic bronchitis has been taking with ephedrine
for a long time. What is the drug‘s action mechanism?
A. Stimulation of noradrenaline release into synaptic
cleft
B. Blockade of noradrenaline release into synaptic cleft
C. Stimulation of (-adrenoceptors
D. Blockade of (-adrenoceptors
E. Direct spasmolytic action

147. Indicate the state, which requires ephedrine introduction:


A. Tachycardia
B. Insomnia
C. Caffeine poisoning
D. Arterial hypotension
E. Arterial hypertension

148. A patient with obstructive bronchitis has been taking ephedrine


for a long time without doctor‘s control. What side effect can be
observed in the patient?
A. Apathy
B. Sleepiness
C. Excitation of the CNS
D. Bradycardia
E. Hypotension

149. Collapse developed in a patient due to decrease of peripheral


vessels tone. Which agent is useful in this situation?
A. Metoprolol
B. Mesaton (Phenylephrine)
C. Xylomethazoline
D. Proserine (Neostigmine)
E. Prasosine

150. A patient with bronchial asthma had been taking orally an agent,
which caused insomnia, headache, increased AP. What medicine can cause
such complication?
A. Ephedrine
B. Isadrinum
C. Prasosine
D. Adrenaline
E. Dopamine

151. The patient suffering from idiopathic hypertension had been


administered the complex treatment. Later he began to complain of
pains in the epigastric region, diarrhea, heartburn. Which agent could
provoke the listed adverse effects in the patient?
A. Sympatholytics (Reserpine)
B. Ganglioblockers (Pentamine)
C. (-adrenoblockers (Anapriline))
D. (1- adrenoblockers (Prasosine)
E. Spasmolytics (Papaverine)

152. A patient who had been suffering from arterial hypertension was
treated with an agent which mechanism of action is connected with
exhaustion of norepinephrine content in sympathetic nerve ending.
Indicate this agent:
A. Clophelinum
B. Anapriline
C. Prasosine
D. Metoprolol
E. Reserpine

153. A patient who has been suffering from severe form of arterial
hypertension after examination was diagnosed pheochromocytoma (tumor
of adrenal medulla which is accompanied by increased synthesis of
epinephrine). Indicate the drug group to treat patient before surgical
treatment:
A. alpha-adrenoblockers
B. Alpha-adrenomimeticks
C. Ganglioblockers
D. Sympathomimetics
E. beta-adrenoblockers

154. A 58-year-old patient with arthritis during treatment with


aspirin complains on nausea, distress in epigastrium. The doctor has
cancelled aspirin and prescribed a NSAIA that is a selective
cyclooxygenase-II inhibitor. Indicate the required agent for the
patient:
A. Naloxone
B. Paracetamol (Acetaminophen)
C. Meloxicam
D. Diclofenac-natrium (Voltaren)
E. Tramadole

155. Which of the following drugs interrupt the cyclooxygenase pathway


of eicosanoid synthesis by nonselective inhibiting both
cyclooxygenase-1 and -2 (COX-1 and -2)?
A.Meloxicam
B.Celecoxib
C.Nimesulide
D.Diclofenac-natrium
E.Allopurinol

156. In addition to providing symptomatic, supportive care, which of


the following drugs would be a helpful adjunct to manage severe
aspirin poisoning?
A. Acetaminophen
B. Acetylscysteine
C. Diazepam
D. Sodium bicarbonate
E. Unitiolum

157. Aspirin causes anti-inflammatory action in result of depression


of cyclooxygenase activity. Level of what biologically active
substances will drop?
A. Biogenic amines
B. Catecholamines
C. Leukotriens
D. Iodthyronines
E. Prostaglandins

158. At Aspirin overdose, method of urine alkalization is used. This


method is directed to:
A. Stimulate a secretion of the acid in proximal tubules
B. Neutralize acid in blood
C. Stimulate reabsorption of the acid in tubules
D. Suppress a secretion of the acid in proximal tubules
E. Suppress reabsorption of the acid in renal tubules

159. Pains in the back developed after the lifting of heavy loads.
Lumbosacral radiculitis was diagnosed. It is known from the anamnesis
that the patient was suffering from ulcer of the duodenal bulb for a
long period of time. Make the most rational choice of nonopioid
analgesics:
A. Butadion
B. Ortophenum (diclofenac sodium)
C. Indomethacin
D. Meloxicam
E. Ibuprofen

160. Gum bleeding arose in the patient after extraction of the tooth.
From anamnesis it was revealed that the patient suffered from
rheumatic arthritis, and was treated with the anti-inflammatory agent
acetyl-salicylic acid (aspirin). Indicate the reason of arisen
bleeding.
A. Suppression of synthesis of uric acid
B. Promotion of thrombolysis
C. Inhibition of hemopoesis
D. Decreasing of blood coagulation
E. Suppression of thromboxane synthesis

161. Utilization of arachidonic acid via cyclooxygenase pathway


results in formation of some bioactive substances. Name them:
A. Biogenic amins
B. Insulin-like growth factors
C. Thyroxine
D. Prostaglandins
E. Somatomedins

162. A patient with rheumatoid arthritis was prescribed a NSAID –


Diclofenac sodium. After a period of time the patient experienced
aggravation of a concomitant disease that forced to withdraw the drug.
What concomitant disease could lead to the drug‘s withdrawal?
A. Diabetes mellitus
B. Ischaemic heart disease
C. Peptic ulcer of a stomach
D. Bronchial asthma
E. Hypertensive disease

163. Aspirin causes anti-inflammatory action in result of depression


of the cyclo-oxygenase activity. Level of what biologically active
substance will drop?
A. Biogenic amines
B. Prostaglandins
C. Iodthyronines
D. Leukotrienes
E. Catecholamines

164. A 60 y.o. patient with rheumatoid polyarthritis has been taking


indometacin for a long time. What is the mechanism of action of this
drug?
A. Blockade of lipoxygenase
B. Blockade of acetylcholinesterase
C. Blockade of cyclooxygenase
D. Blockade phosphodiesterase
E. Blockade of phospholipase

165. A patient with severe, acute trauma pain requires analgesia. The
physician orders morphine. Which of the following coexisting
conditions would pose the greatest risk from morphine‘s use in this
case?
Acute pulmonary edema
Hypertension
Closed head injury
Opioid abuse
Recent myocardial infarction

166. At postoperative period a patient has been receiving Promedol for


a long time. After a cancellation of the drug the patient developed
serious mental, neurological and somatic disturbances. Define this
symptom-complex.
A. Idiosyncrasy
B. Rebound syndrome
C. Tachyphylaxis
D. ‗Stealing‘ syndrome
E. Abstinent syndrome

167. To reduce the pain at traumatic shock Morphine hydrochloride has


been given. What is the principal mechanism of analgesic action of it?
R
Block of peripheral sensation receptors
Inhibition of prostaglandins synthesis
Blockade of opioid receptors
Stimulation of opioid receptors
Blockade of adrenoreceptors

168. A synthetic analgesic, which increases myometrium contraction and


relax the neck of uterus is administered to a woman in labor. Indicate
this agent.
A. Omnoponum
B. Promedol (Trimeperidine)
C. Morphine
D. Fentanyl
E. Fenoterol

169. A patient has signs of acute poisoning with morphine: sharp


miosis, loss of consciousness, decrease of BP and Cheyne-Stokes
respiration. Administer the pharmacological antagonist:
A. Nalorphine
B. Promedolum
C. Pentazocinum
D. Naloxone
E. Tramadol

170. These biological active substances in the CNS have analgesic


activity and are similar to morphine. Determine this active substance.
A. Opiates
B. GABA
C. Beta-endorphin
D. Cyclooxygenase
E. Dopamine

171. The patient was prescribed a narcotic analgesic known as a


derivative of cyclogexanol, agonist-antagonist of opioid receptors
that is weaker than morphine. What is the agent?
A. Naltrexone
B. Fentanyl
C. Tramadol hydrochloride
D. Codeine phosphate
E. Naloxone

172. Why morphine is undesirable to use in patients with cardio


cerebral trauma?
A. Provokes cardiac arrhythmias
B. Reduces blood pressure
C. Increases intracranial pressure
D. Causes psychological and physiological dependence
E. Suppresses respiration

173. At admission department it has been delivered a 30-year-old man


after car accident with hip fracture. He has decreased BP, thread-like
pulse, local tenderness of the broken hip. What is necessary to give
to the patient for traumatic shock prevention?
A. Tramadol
B. Naltrexone
C. Fentanyl
D. Naloxone
E. Aspirin

174. The patient has receiving morphine for a long time because of
breast cancer. What does not belong to the pharmacological effects of
morphine?
Constipation
Inhibition of respiratory center
Spasm of Oddi sphincter
Decrease of rate of respiration
Mydriasis

175. Examination of a patient revealed extremely myotic pupils,


sleepiness, infrequent Chain-Stoke‘s respiration, urinary retention,
slowing-down of heart rate, and enhancement of spinal reflexes. What
substance caused the poisoning?
A. Morphine
B. Atropine
C. Barbital
D. Caffeine
E. Phosphacole

176. A 52-yars –old patient who had been suffering from urolithiasis
was delivered to the emergency department with renal colic. A doctor
administered atropine together with opioid analgesic with spasmolytic
activity to prevent development of the pain shock. Choose this drug.
A. Tramadol
B. Promedol (Trimeperidine)
C. Triftazinum (trifluoperazine)
D. Vasopressin
E. Sulpiride

177. Morphine has been administered to reduce the pain at traumatic


shock. What is the analgesic action mechanism of the drug?
A. Blockade of Na+ channels
B. Blockade of peripheral sensation receptors
C. Stimulation of opioid receptors
D. Inhibition of cyclooxygenase
E. Inhibition of prostaglandin synthesis

178. A 42 year-old man who has been injured in a car accident is


brought to the emergency room. His blood alcohol level on admission is
250 mg/dL. Hospital records show a prior hospitalization for alcohol
related seizures. His wife confirms that he has been drinking heavily
for 3 weeks.
What treatment should be provided to the patient if he goes into
withdrawal?
Diazepam
Phenobarbital
Pentobarbital
Diphenin (Phenytoin)
None
179. A patient of 17 years addressed a doctor with complaints of
insomnia manifested in hard falling asleep that led to fatigue,
weakness, difficulty of learning. The clinical examination revealed
the following: irritability, emotional instability, pulse and BP
alteration. The doctor determined that insomnia was associated with
neurosis-like state and vegetovascular distonia. Choose the most
rational agent for correction of this condition.
A. Droperidol
B. Nitrazepam
C. Aminazine (Chlorpromazine)
D. Phenobarbital
E. Galoperidolum

180. A patient has been taking a mixture prescribed by


neuropathologist for neurasthenia for a week. The patient feels
better, but has developed conjunctivitis, rash, inertia, decrease of
memory. She is diagnosed bromism. What should be prescribed to
decrease the symptoms?
Naloxone
Atropin
Panangin
Natrium Chloride
Pilocarpine

181. Choose drug combination producing neuroleptanalgesia.


A. Haloperidole+Paracetamole
B. Droperidole+Naloxone
C. Diclofenac-natrium+Haloperidole
D. Droperidole + Fentanylum (Innovar)
E. Aminazine+ Nimesulide

182. In a psychiatric clinic during psychosis treatment the symptoms


of medicinal Parkinsonism had appeared. What drug has been used at the
treatment of psychosis?
A. Lithium carbonate
B. Clozapine
C. Diazepam
D. Aminazine (Chlorpromazine)
E. Sulpiride (Eglonil)

183. A patient of 40 years is delivered in psychiatric clinic in a


state of exaltation, aggression, delirium. What drug should be entered
to the patient?
A. Reserpine
B. Aminazine (Chlorpromazine)
C. Diazepam
D. Diclofenac sodium
E. Sodium oxybutiras

184. A patient, getting treatment for neurosis with Sibason


(Diazepam), is complaining on a toothache. A doctor prescribed an
analgesic in the dose less than therapeutic average dose. What
phenomenon doctor took in consideration while decreasing agent‘s dose?
A. Summation
B. Cumulation
C. Tolerance
D. Additive effect
E. Potentiation

185. A woman with active life-style addressed to outpatient department


complaining on bad mood, migraine, emotional liability, and chest
pain. What is wise to administer for neurosis treatment taking into
account that she spends a lot of time at work?
A. Mezapam
B. Phenazepam
C. Triftazine
D. Phenobarbital
E. Aminazine (Chlorpromazine)

186. A patient had been suffering from schizophrenia accompanied by


arterial hypertension. A doctor administered neuroleptic possessing
expressed hypotensive activity. Indicate this drug.
A. Haloperidol
B. Risperidone
C. Aminazine (Chlorpromazine)
D. Diazepam
Triftazinum (trifluoperazine)

187. A patient took a drug from the group of Benzodiazepines for the
treatment of neurosis for a long period of time. One day the patient
felt sleepiness, weakness, and decrease of memory, concentration and
reactions, headache, nausea. The patient was addressed to the
polyclinic and a doctor administered an injection of an antagonist of
the Benzodiazepines. Determine this drug.
A. Bemegride
B. Naltrexone
C. Naloxone
D. Flumazenil
E. Cordiamin
188. What group of drugs is used for the treatment and prevention of
manias?
A. Antidepressants
B. Sedatives
C. Neuroleptics
D. Tranquilizers
E. Lithium

189. Long application of the neuroleptics majority of which invokes


and enhances symptoms of Parkinsonism, is necessary for the patient
with pre-existing slight parkinsonism syndrome. What neuroleptic is
least dangerous in that case?
A. Aminazine (Chlorpromazine)
B. Droperidole
C. Triftazine
D. Clozapine (Leponex)
E. Haloperidol

190. Aminazine was administered to a patient with schizophrenia. Which


pharmacodynemic effect of aminazine is the basic for this patient?
A. Anxiolytic
B. Antipsychotic
C. Myorelaxant
D. Hypotensive
E. Hypothermic

191. A patient of 28 years in a state of exaltation, aggression,


delirium is delivered to psychiatric clinic. What drug should be
administered to the patient?
A. Diazepam
B. Sodium bromide
C. Tincture of Valerian
D. Aminazine (Chlorpromazine)
E. Lorazepam

192. For performing an operative measure, it is necessary to use a


method of general hypothermia. At application of which agent in
combination with physical refrigerating the expressed hypothermia is
observed?
A. Diazepam
B. Sodium bromide
C. Tincture of Valerian
D. Aminazine (Chlorpromazine)
E. Lorazepam
193. Glutamate decarboxylation results in formation of inhibitory
transmitter in CNS. Name it:
A. Serotonin
B. GABA
C. Glutathione
D. Histamine
E. Asparagines

194. A 35 y.o. patient with schizophrenia complains on discoordination


of movement, tremor of hands, drowsiness. The patient has been taking
psychotropic agent for a long time. Which agents are most likely to
cause these disturbances?
A. Tranquilizers
B. Atypical neuroleptics
C. Typical neuroleptics
D. Antidepressants
E. Psychostimulants

195. Phenazepam has been prescribed to a patient with hyper


excitability, irritability, tearfulness, sleeplessness. What is the
drug action mechanism?
A. Blockade of GABA-receptors
B. Stimulation of benzodiazepine receptors
C. Blockade of M-cholinoceptors
D. Blockade of phosphodiesterase
E. Stimulation of dopamine receptors

196. For the treatment of delirium and hallucination Aminazine


(Chlorpromazine) has been prescribed. What is its antipsychotic action
mechanism?
A. Blockade of reuptake of catecholamines
B. Stimulation of adrenergic and dopaminergic processes in the CNS
C. Inhibition of adrenergic and dopaminergic processes
in the CNS
D. Blockade of M-cholinoceptors
E. Stimulation of M-cholinoceptors

197. A doctor administered Aminazinum (chlorpromazine) to the patient


suffering from schizophrenia to eliminate delirium, hallucinations, to
decrease aggression and psychomotor excitement. What is the mechanism
of antipsychotic action of Aminazinum?
A. Excitation of M-cholinoreceptors in the CNS
B. Stimulation of opioid reeptors
C. Blockade of D2-dopamine receptors in the the CNS
D. Excitement of adrenoreceptors and dopamine receptors in the CNS
E. Inhibition of MAO
198. A 32 y.o. woman was addressed to a doctor with complaints of
temper, tiredness, insomnia, internal tension. The doctor had
diagnosed neurosis and administered tranqilizer (diazepam). Which of
the effects of this agent is more important in this situation?
A. Antipsychotic
B. Anxiolytic
C. Anticonvulsive
D. Myorelaxant
E. Antiemetic

199. A dentist introduced sibazonum (diazepam) to a 47 y.o. woman


before extraction of tooth. Indicate the anxiolytic action mechanism
of the drug:
A. Stimulation of opioid receptors
B. Inhibition of dopamine receptors and adrenoceptors
C. Stimulation dopamine receptors and adrenoceptors
D. Agonist of benzodiazepine receptors
E. Inhibition of benzodiazepine receptors

200. A patient was addressed to a doctor with complaints of


irritability, insomnia, fatigue. A doctor administered a sedative drug
to him. In a week the patient began to complain of cough, sleepiness,
decrease of memory, phenomena of rhinitis, conjunctivitis, dermatitis.
Which group of the drugs was administered by the doctor?
A. Neuroleptics
B. Tranquilizers
C. Lithium
D. Bromides
E. Non-steroidal anti-inflammatory agents

201. Drug of choice for psychomotor epilepsy is:


A. Valproic acid
B. Carbamazepine
C. Ethosuximide
D. Barbiturate
E. Diphenin

202. Which anti-epileptic acts by augmentation of release of


inhibitory transmitter GABA by inhibiting its degradation (by GABA-
transaminase) as well as probably by increasing its synthesis?
A. Valproic acid
B. Carbamazepine
C. Diphenin (Phenytoin)
D. Ethosuximide
E. Barbiturates

203. Which anti-epileptic acts by prolongation of Na+ channels


inactivation as well as by inhibiting kindling? The drug also has
antidiuretic action, probably by enhancing ADH action on renal
tubules.
A. Carbamazepine
B. Phenobarbital
C. Droperidole
D. Valproate sodium
E. Ethosuximide

204. A major problem that must be faced when administering


anticonvulsants with many other medications (including other
antiepileptic drugs) involves drug interactions due to altered
metabolism. Which of the following drugs is likely to cause excessive
or toxic effects from some other drugs by inhibiting metabolism?
A. Ethosuximide
B. Carbamazepine
C. Phenobarbital
D. Phenytoin
E. Valproic acid
205. A man develops akathisia,a Parkinson-like syndrome, galactorrhea,
and amenorrhea, during the therapy. Which of the following mechanisms
is present?
A.Blockade of muscarinic receptors
B. Blockade of dopamine receptors
C. Blockade of +-adrenergic receptors
D. Stimulation of dopamine receptors
E. Supersensitivity of dopamine receptors

206. The doctor has prescribed Cyclodolum for Parkinson disease


treatment. What is the mechanism of anti parkinsonian action of the
drug?
A. M cholinomimetic action
B. M cholinoblocker action
C. Stimulation of dopamine receptors
D. Blockade of dopamine receptors
E. (-Adrenoblocker action

207. A patient of 58 year old has been taken Phenobarbital in


connection with insomnia. After a cancellation of the drug the patient
again has difficulty in falling asleep, frequently wakes up at night;
dreams is accompanied by dreadful dreams. What is the reason of the
given undesirable effect of Phenobarbital?
A. Dependence
B. Idiosyncrasy
C. Tachyphylaxis
D. Rebound syndrome
E. Tolerance

208. A patent defied doctor‘s instructions and continued to use


Phenobarbital over 3 weeks. Meanwhile, he was forced to increase the
drug‘s dose. How to explain falling of Phenobarbital efficacy?
A. Activation of lipolysis
B. Inhibition of monooxygenase systems of a liver
C. Induction of monooxygenase systems of a liver
D. Inhibition of lipolysis
E. Induction of glycolysis

209. A 56 y.o. man with insomnia was given Phenobarbital. The sleep
was normalized. However, after 2 weeks the effect of Phenobarbital
fell down. What reason caused decreasing of hypnotic action of
Phenobarbital?
A. Its poor solubility
B. Development of tolerance
C. Its poor absorption in stomach
D. Its fixation in lipids
E. Development of tachyphylaxis

210. A patient with epilepsy was recommended a medicine known as a


derivative of dipropilacetic acid that inhibits GABA-transferase and
increases GABA level in the brain; inhibits excitability and seizure
capability of motor zone of the CNS. It can be used for all types of
epilepsy. Name this medicine:
A. Reserpine
B. Diazepam (Seduxen)
C. Diphenin
D. Sodium bromide
E. Sodium valproate

211. Prescribe a drug that belongs to modern hypnotics to the patient


with insomnia. It is an imidazopyridine derivative; activates
benzodiazepine receptors in the CNS; does not alter either sleep
structure or activity of liver enzymes, does not cause dependence.
A. Droperidol
B. Nitrazepam
C. Zolpidem
D. Chloral hydrate
E. Flumazenil
212. What phenomena may be developed in long taking of barbiturates?
Drug tolerance
Drug dependence
Extrapyramidal syndrome
Sensibilization
Inhibition of enzymes of liver

213. The patient has appealed to the dentist with complaints of


hypertrophy and pain in the gums. After examination of the oral cavity
the doctor has diagnosed hyper plastic gingivitis. It has been
established from the anamnesis, that the patient has been taking for a
long time an antiepileptic agent. Specify this drug.
A. Diphenin (Phenytoin)
B. Hexamidinum
C. Phenobarbital
D. Carbamazepine
E. Trimethinum (trimethadione)

214. A patient has trauma-caused brain edema and hypoxic cramps. What
agent is rational to use for abortion of the cramps?
A. Cordiaminum
B. Anaprilinum (Propranolol)
C. Bemegride
D. Sodium oxybutyras
E. Piracetam

215. A 57 y.o. man in a coma has been delivered to hospital. It is


known from his anamnesis that he suffered from insomnia. At inspection
the following is revealed: respiratory depression, fallen AP,
progressing cardiac weakness, decrease of body to, inhibition of
tendon reflexes. What drug could cause poisoning?
A. Fluoxetine
B. Phenobarbital
C. Tincture of Valerian
D. Levodopa
E. Sodium bromide

216. A woman after the experienced nervous shock badly sleeps. What
agent should be administered?
A. Fluoxetine
B. Phenobarbital
C. Nitrazepam
D. Aminazine (Chlorpromazine)
E. Sodium bromide
217. Diphenin (Phenytoin) has been administered to a patient for
epilepsy with tonic-clonic attacks. What is the action mechanism of
the drug?
A. Activation of GABA system
B. Blockade of Ca2+ channels
C. Stimulation of dopamine receptors
D. Blockade of dopamine receptors
E. Blockade of Na+ channels

218. A 76 y.o. patient has appealed to the doctor with complaints of


superficial short-term sleep with often awakenings caused by sense of
internal tension, anxiety, fear. Senile sleeplessness was diagnosed.
Make a rational choice of a hypnotic in the given situation:
A. Phenobarbital
B. Chloral hydrate
C. Nitrazepam
D. Lamotrigine
E. Carbamazepine

219. A patient with convulsions was delivered to the hospital where


status epilepticus was diagnosed. Indicate the first choice drug to
treat the patient:
A. Carbamazepine
B. Diazepam
C. Phenobarbital
D. Levodopa
E. Chloral hydrate

220. An attack of generalized tonic-clonic convulsions accompanied by


loss of consciousness and general suppression of the CNS developed in
a patient after trauma. Which agent should be administered to this
patient?
A. Cyclodol (Trihexyphenidyl)
B. Midantan (Amantadine)
C. Levodopa
D. Soidium oxybutiras
E. Phenobarbital

221. Symptoms of poisoning by derivative of barbituric acid are


revealed in a patient. What drug can increase excretion of
barbiturates from the organism?
A. Natrium chloride
B. Natrium hydrocarbonate
C. Kalium chloride
D. Magnesium oxide
E. Magnesium sulfate

222. Determine the hypnotic drug, which doesn‘t influence the


structure of sleep.
A. Barbital
B. Phenobarbital
C. Nitrazepam
D. Aethaminalum-natrium (pentobarbital)
E. Bromisoval

223. MAO inhibitors are contraindicated with all of the following


EXCEPT:
A. Indirect adrenergic agents, such as ephedrine
B. Tricyclic antidepressant
C. Paracetamol
D. Beer and cheese
E. Dopamine

224. The patient addressed to a doctor with complaints of bad mood,


that was accompanied by expressed sedative action, feeling of fear,
anxiety. The clinical examination revealed psychical depression. What
drug should be administered to this patient?
Caffeine
Sulfocamphocaine
Pyracetam
Amitriptyline
Droperidol

225. A doctor administered amitriptyline to a patient with to a


patient endogenous depression. Explain the mechanism of action of this
drug.
A. Inhibition of the neuronal reuptake of noradrenaline
B. Inhibition of the neuronal reuptake of serotonin
C. MAO inhibitor
D. Increase of release of noradrenaline and serotonin
E. Inhibition of the neuronal reuptake of noradrenaline
and serotonin

226. A man was addressed to the psychiatrist with complaints and


desperation, tendency to suicide. Determine the group of drugs for the
treatment of this patient?
A. Antidepressants
B. Sedatives
C. Neuroleptics
D. Tranquilizers
E. Lithium

227. This group of drug includes psychotropic agents of plant origin.


These drugs are used in asthenic states after severe infectious
diseases. They increase general vital tonicity of the organism and its
resistance to infectious diseases. These drugs are used in the form of
tinctures or liquid extracts. Choose this group of drug.
A.Sedative
B.Adaptogens
C. Psychostimulants
D.Antidepressants
E.Nootrop agents

228. This drug has stimulating action promoting synthesis of proteins


and ATP, increases the capacity for physical and psychical work. It is
used for a long time and it shouldn‘t be taken before sleep.
Tincture of Ginseng
Tincture of Valerian
Barboval
Nialamide
Natrium bromide

229. Analeptic of reflective type from the N-cholinomimetics group was


given to the patient for restoration of breathing after poisoning with
carbon monoxide. What agent was administered to the patient?
Adrenaline hydrochloride
Lobeline
Mesaton
Naloxone
Caffeine

230. Indicate the mechanism of action of Caffeine.


A. Blockade of Monoaminooxidase (MAO)
B. Inhibition of the neuronal capture of Noradrenaline
C. Inhibition of Phosphodiesterase (PDE)
D. Blockade of Adenosine receptors.
E. Blockade of adenosine receptor and inhibition of
phosphodiesterase

231. Indicate the condition for the use of Piracetam:


A. Significant atherosclerosis
B. Mental insufficiency
C. Convulsions
D. Hypertensive crisis
E. Convulsions

232. Indicate the analeptic with the mixed type of action:


A. Cordiamin
B. Caffeine
C. Lobeline hydrochloride
D. Bemegride
E. Aethimizolum

233. Indicate the analeptic with the direct type of action:


A. Cordiamin
B. Cytiton
C. Lobeline
D. Bemegride
E. Sulfocamphocaine

234. During the operation under general anesthesia the patient‘s


respiration was inhibited stimulant should be used in this stimulant
without pausing the general anesthesia.
A. Aethymisole
B. Caffeine
C. Bemegride
D. Cytiton
E. Lobeline hydrochloride

235. Indicate the drug which possesses analeptic and psycho stimulant
activity.
A. Caffeine
B. Bemegride
C. Aethimizolum
D. Cordiaminum (nikethamide)
E. Strychnine

236. Specify application for use of Bemegride:


A. Parkinsonism
B. Hypertensive crises
C. Overdose of general anesthetics
D. Brain hypoxia
E. Tachyarrythmias

237. The patient was admitted to the neurology department because of


complaints of decrease of memory, mental and work capacity,
sleeplessness and vertigo. His symptoms were connected to a brain
concussion, which took place 2 years ago as a result of an automobile
accident. What drug should be indicated to improve his condition?
A. Pyracetam (Nootropil)
B. Natrii oxybutiras (oxybyte sodium)
C. Sydnocarb
D. Caffeine
E Cordiamin (nikethamide)

238. A 28 y.o. patient was delivered to the intensive care unit with
the diagnosis of poisoning with barbiturates of mild degree.
Administer a stimulant of respiration, which will be pertinent in this
case:
A. Instenon
B. Bemegride
C. Sydnocarb
D. Piracetam
E. Diazepam

239. Thiopental natrium was introduced to the patient for initial


general anesthesia that caused the arterial hypotension and inhibition
of breathing. Select a drug used for restoration of breathing and
blood circulation.
A. Lobeline
B. Dipiroxim
C. Naloxone
D. Cytiton
E. Bemegride

240. The child was born in a state of asphyxia. What drug is necessary
to administer to the newborn for stimulation of respiration?
A.Proserine (Neostigmine)
B.Naloxone
C.Prazosin
D.Atropine
E.Etimizol

241. A patient with major depression has been prescribed Nialamide.


The doctor has warned the patient about necessity to exclude from diet
during the treatment:
A.Cheese
B.Apples
C.Potato
D.Cabbages
E.Pears
242. To the patient with a bipolar manic-depressive illness in a stage
of depression, which complained of feeling of alarm, fear, the
antidepressant with concomitant sedative effect has been prescribed.
What was a drug?
A. Nialamide
B.Imizine (Imipramine)
C.Sydnocarb
D.Amitriptyline
E.Levodopa

243. A patient after cranial trauma was delivered to the neurological


department. Progressive increase of neurological symptoms allowed
making the diagnosis of brain contusion, which became the reason of
increasing edema of the brain followed by hypoxia of its structures.
Administer to the patient a general anesthetic, which exerts
beneficial influence in brain hypoxia:
A. Phthorotan
B. Isofluran
C. Propanidide (Sombrevine)
D. Ketamine
E. Natrii oxybutiras (GOBA)

244. In severe alcohol intoxication, the blood ethanol exceeds:


A. 0.5 g/L
B. 1 g/L
C. 2 g/L
D. 3 g/L
E. 4 g/L

245. The same dose of alcohol invokes a different degree of alcohol


intoxication in different people. It is linked with:
A. High plasma level of beta-lipoproteins
B. Inhibition of insulin synthesis
C. Genetically determined insufficiency of alcohol
dehydrogenase synthesis
D. Inhibition of hepatic microsomal enzymes
E. Low activity of pseudocholinesterase (butyrylcholinesterase)

246. To anaesthetize the manipulation related to a burned surface


treatment, a patient was IV injected a medication for short-acting
narcosis. 1 minute later the patient being under anesthesia had an
increased BP, tachycardia, increased tone of skeletal muscles;
reflexes remained. After awakening the patient had disorientation and
visual hallucinations. What medication was the patient injected?
A. Nitrous oxide
B Thiopental sodium
C Diethyl ether
D Sombrevin
E Ketamine

247. A patient is 25 years old with a poly trauma, unconscious, the AP


is 95/70 mm Hg, pulse is 75 beats per minute, respiration superficial
with frequency 30 in one min. What agent should be selected for a
narcosis (general anesthesia)?
A. Ketamine
B. Thiopental sodium
C. Diethyl ether
D. Sombrevin
E. Fluorotane (Halothane)

248. Thiopental sodium was IV administered to a patient for an


introduction narcosis and then the laryngospasm and hypersalivation
have developed. Administration of what drug could prevent undesirable
effects?
A. Nitrazepam
B. Carbamazepine
C. Alloxime
D. Atropine
E. Flumazenil

249. With the purpose of warming after cooling, the patient used
solution of ethanol.How does ethanol influence on thermoregulation?
A. Increases heat release
B. Decreases heat release
C.Increase heat production
D. Decreases heat production
E. Decreases heat production and heat release

250. The doctor has prescribed teturam (disulfiram) for the treatment
of the patient with alcoholism. What is the mechanism of action of the
given drug?
A. Activation of vomiting Center
B. Augmentation of Acetylcholinesterase
C. Inhibition of Acetylcholinesterase
D. Augmentation of aldehyde dehydrogenase activity
E. Inhibition of Aldehyde Dehydrogenase

251. A 15-years-old boy was admitted to the hospital with polytrauma


in unconscious state, BP 95/70 mm Hg, heart rate 72 per minute,
superficial breathing (respiration rate is 30 per minute). Choose the
drug for general anaesthesia.
A. Propanidid
B. Hexenalum (hexobarbital)
C. Ketamine
D. Thiopental natrium
E. Phthorotanum (halothane)

252. General anesthetic with ultra short action (5 minutes) was


introduced to a patient intravenously for analgesia of biopsy. It
caused muscular twitching, insignificant decreased of blood pressure,
stop of breathing for short period of time. Determine this drug.
A. Ketamine
B. Natrii oxybutiras
C. Pentazocinum
D. Propanidid
E. Phthorotanum (halothane)

253. Propanidid is used very often in dentist practice. Determine the


contraindication for use of this drug.
A. Reposition of jawbone fragments
B. Shock
C. Reposition of dislocations of jaw
D. Taking out the stitches
E. Dissection of pulp cavity

254. The patient used solution of ethyl alcohol for warming after
super cooling. How does ethyl alcohol influence upon thermoregulation?
A. Decreases heat production and heat emission
B. Decreases heat emission
C. Increases heat production
D. Decreases heat production
E. Increases heat emission

255. Expressed bradycardia, reducing of AP appeared during


phthorotanum anesthesia. What drug is contraindicated to normalize the
heart rate for continuation of the operation?
A. Proserpine (neostigmine)
B. Atropine
C. Mesatonum (Phenylephrine)
D. Caffeine
E. Adrenaline (epinephrine)

256. A patient suffers from chronic alcoholism with the following


symptoms: pain in arms and legs, impairment of skin sensitivity,
muscle weakness, edemas and increased amount of private. Which vitamin
drug should be prescribed to the patient?
A. Ergocalcipherol
B. Thiamine
C. Retinal
D. Rutin
E. Vikasolum (Menadione)

257. A 35 y.o. patient who often consumes alcohol was treated with
diuretics. There appeared serious muscle and heart weakness, vomiting,
diarrhea. AP – 100/60 mm Hg, depression. This condition is caused by
intensified excretion with of:
A. Calcium
B. Potassium
C. Chlorine
D. Phosphates
E. Sodium

258. A patient, who has been treating with Clophelinum (Clonidine) due
to arterial hypertension, has taken an alcoholic drink that has caused
the sharp inhibition of the CNS. What is the name of this interaction?
A. Potentiation
B. Summation
C. Cumulation
D. Intoxication
E. Idiosyncratic reaction

259. A sudden acute decrease of arterial pressure arise in a patient


who was being operated under general anesthesia. The doctor introduced
epinephrine following by ventricle fibrillation. Indicate general
anesthetic, which can most likely induce this complication:
A. Nitric oxide
B. Propanidide
C. Phthorotanum (Halothane)
D. Ketamine
E. Thiopental-sodium

260. A patient of 50 years old with complaints of heart palpitation,


pain in the heart area, headache, and vertigo was delivered to a
hospital. During the examination angina pectoris, disorders of cardiac
rhythm (paroxysmal form of atrium fibrillation) and arterial
hypertension (180/90 mm Hg) were revealed. Make the rational choice of
the drug for this patient:
A. Propranolol
B. Epinephrine
C. Enalapril
D. Nifedipine
E. Digoxin
261. During the ambulant tooth extraction a patient who had been
suffering from chronic congestive heart failure showed the typical
signs of acute heart failure: dyspnea, cyanosis, tachycardia, ps 96
per minute, BP 100/60 mm Hg. Indicate the drugs to be used to treat
this patient.
A. Sthrophanthine + Verospirone
B. Lisinopril + Furosemide
C. Anapriline + Sulfocamphocaine
D. Sthrophanthine + Furosemide
E. Anapriline + Furosemide

262. A 35-y.o. male has undergone surgery for necrotic bowel. Despite
having been treated with antibiotics, on the 5th postoperative day, he
develops symptoms (fever, hypotension, tachycardia, declining urine
output, confusion) consistent with septic shock. What hemodynamic
support would be helpful?
A. Fluid administration
B. Dobutamine infusion
C. Fluid and dobutamine infusion
D. Atropine administration
E. Antibiotic administration

263. The action of electric current on the excitable myocardial cell


caused Depolarization of its membrane. Movement of what ions through
the membrane caused depolarization?
A. Ca2+
B. K+
C. Cl-
D. Na+
E. HCO3

264. Treatment of Digitals toxicity can include all except:


A. Potassium
B. Phenytoin
C. Lidocaine
D. Dialysis
E. Unitiolum

265. A 45 y.o. patient has been prescribed Digoxin to treat chronic


heart failure. What is the mechanism of action of Digitoxin?
A. Inhibition of Na+/K+- ATPase
B. Decreases intracellular Na+ concentration
C. Increases the intracellular level of ATP
D. Stimulates production of cAMP
E. Decreases release of Ca2+ from the sarcoplasmic reticulum
266. A patient with a cardiac arrhythmia is being treated for a long
time with Amiodarone. This drug can cause biochemical changes and
clinical signs and symptoms that resemble those associate with which
of the following disease?
A. Diabetes mellitus
B. Addisonian crisis
C. Hypothyroidism
D. Diabetes insipidus
E. Cushing‘s syndrome

267. Which antiarrhythmic drug has relatively few electrophysiologic


effects on normal myocardial tissues, but suppresses the
arrhythmogenic properties of ischemic tissues?
A. Digoxin
B. Propranolol
C. Lidocaine
D. Amioadrone
E. Phenitoin

268. Specify the mechanism of antiarrhythmical action of Amiodaronum


(Cordaronum):
A. Blockade of Na+ channels of the cardiomyocyte membranes
B. Blockade of Ca2+ channels of the cardiomyocyte membranes
C. Blockade of beta – adrenoceptors of the myocardium
D. Blockade of alfa – adrenoceptors of the myocardium
E. Blockade of K+ channels of the cardiomyocyte
membranes

269. Antiarrhythmic with greatest repolarization phase is:


A. Quinidine
B. Amiodarone
C. Lidocaine
D. Propranolol
E. Verapamile

270. In terms of therapeutic usefulness, the most important


pharmacologic action of digoxin in congestive heart failure is:
A. The reduction of cardiac size
B. The increase in ventricular contractile force
C. The slowing of heart rate
D. The diuretic effect
E. The increase in blood pressure
271. The patient with a cardiogenic shock has been delivered into an
emergency room. Choose a drug of a first choice:
A. Dopamine
B. Propranolol
C. Cordiamine
D. Adrenaline hydrochloride
E. Enalapril

272. The drug of choice in anaphylactic shock is:


A. Dopamine
B. Propranolol
C. Cordiamine
D. Adrenaline hydrochloride
E. Enalapril

273. The half life of digoxin is:


A. 12 hrs
B. 24 hrs
C. 36 hrs
D. 5 days
E. 2 weeks

274. In a clinic, the patient complained of unpleasant sensations in


the heart region, and attacks of weakness and loss of consciousness.
Inspection of the patient‘s electrocardiogram had revealed the
presence of II degree atrioventricular block. Specify a drug, which
should be used in the situation:
A. Isadrinum
B. Novocainamide
C. Nitroglycerine
D. Strophanthin
E. Anaplilinum

275. A 50 y.o. patient with chronic cardiac insufficiency and


tachyarrythmia was prescribed a cardiotonic drug. What drug was
prescribed?
A. Dobutamine
B. Amyodarone
C. Dopamine
D. Digoxin
E. Mildronate

276. Digitoxin (1 tab tid) had been prescribed to a patient with


chronic (congestive) heart failure. In 15 days, the signs of
intoxication developed in the patient – bradyarrhythmia, disturbance
of atrioventricular conduction, upset of colour vision. What could
cause the specified phenomena?
A. Functional cumulation
B. Material cumulation
C. Potentiation
D. Tolerance
E. Summation

277. In homozygous mice with defect of the gene-coding frame of a


beta1—adrenoreceptors, it has been probed action of an agent on
inotropic function of heart. What agent will lose the activity in
these conditions?
A. Dobutamine
B. Mesaton (Phenylephrine)
C. Strophanthin
D. Caffeine
E. Digoxin

278. A water-soluble cardiac glycoside has been prescribed to a


patient. What concomitant pathology can promote a cumulation of water-
soluble cardiac glycosides?
A. Renal failure
B. Liver failure
C. Hyperacide gastritis
D. Hypoacidic gastritis
E. Hypothyroidusm

279. Indicate the mechanism of anti arrhythmic action of Quinidine


sulphate.
A. Blockade of Na+ channels of the cardiomyocyte
membranes
B. Blockade of Ca2+ channels of the cardiomyocyte myocardium
C. Blockade of beta – adrenoceptors of the myocardium
D. Blockade of alfa – and beta – adrenoceptors of the myocardium
E. Blockade of M-cholinoceptors of the myocardium

280. Indicate the group of drugs, overdosage of which is accompanied


by following signs: nausea, vomiting, diarrhea, infringement of heart
activity (extrasystoles, delay of antrioventricular conductivity),
headache, vision impairment (xanthopsia, diplopia).
A. Ca2+ - channels blockers
B. Organic nitrates
C. Cardiac glycosides
D. Beta - adrenoblockers
E. Angiotensin converting enzyme inhibitors
281. A patient suffers from allergic reaction to iodine. Indicate an
anti-arrhythmic agent, which is contraindicated to him.
A. Amiodarone
B. Verapamil
C. Novocainamidum (Procainamide)
D. Ornidum (bretylium)
E. Quinidine sulfate

282. Indicate the drug and its group, which is used for treatment of
atrioventricular blockade.
A Cardiac glycoside Digitoxin
B. Beta-adrenomimetic Isadrinum (isoprenaline)
C. Sympatholytic Ornidum (bretylium)
D. Beta –adrenoblocker Anaprilinum (Propranolol)
E. Blocker of slow calcium channels Verapamil

283. Indicate the group of drugs used to treat total atrioventricular


block:
A. M-cholinoblockers
B. Membrane-stabilizing agents
C. Local anesthetics
D. Beta –adrenoblocker
E. Potassium channels blockers

284. Symptoms of cardiac glycosides toxicity have been developed in a


patient with chronic congestive heart failure. What agent should be
administered to decrease adverse effects of cardiac glycosides?
A. Ethimisol
B. Dipiroxim
C. Potassium chloride
D. Atropine sulfate
E. Novocainamid

285. Owing to a myocardial infarction, the patient had ventricular


arrhythmia. The cardiac rhythm was normalized after introduction of an
antiarrhythmic agent with local anesthetic activity. What drug was
introduced?
A. Verapamil
B. Chinidinum (Quinidine)
C. Sotalol
D. Propranopol
E. Lidocaine
286. An atrioventricular blockade has been revealed in a 25y.o man
during inspection. Name the agent used for treatment of
atrioventricular blockade:
A. Amiodaron
B. Proserinum (Neostigmine)
C. Verapamil
D. Metoprolol
E.Isadrinum

287. A patient has an attack of tachycardia. What type of membrane


receptors of cardiomyocytes is wise to block for stopping of the
attack?
A. M-cholinoreceptors
B. Beta1 – adrenoceptors
C. Beta2 – adrenoceptors
D. Alfa- adrenoceptors
E. M-and N- cholinoreceptors

288. For correction of the arrhythmias caused by poisoning with


cardiac glycosides, the antiepileptic agent diphenin (phenytoin) can
be used. What effect is typical for diphenin (phenytoin)?
A Quickens repolarization and reduces the efficient
refractory period
B. Blocks beta- adrenoceptors
C. Decreases contractility of myocardium
D. Suppresses contractility of heart
E. Prolongs Phase 3 repolarisation

289. The permeability of the irritable cell membrane has been


increased for potassium ions during an experiment. What changes of
membrane electric status can occur?
A. Depolarization
B. Action potential
C. No changes
D. Local response
E. Hyperpolarization

290. A patient with chronic cardiac insufficiency has been treated


with cardiotonic drugs and a thiazide diuretic, but in spite of it
there are still edemata and risk of ascites. What medication should be
prescribed to amplify diuretic effect of the applied drugs?
A. Spironolactone
B. Furosemide
C. Manitole
D. Amiliride
E. Clopamide
291. Which of the following drugs is appropriate and generally
regarded as most effective for relieving and preventing ischemic
episodes in patients with variant angina?
A. Aspirin
B. Lovastatine
C. Propranolol
D. Diltiazem
E. Nitroglycerin

292. Nitrites used in management of angina have all the following


action except:
A. Increase total coronary blood flow
B. Decrease cardiac O2 consumption
C. Decrease preload and after load
D. Redistribution of blood into subendocardial vessels
E. Dilatation of cerebral vessels

293. In a patient with angina pectoris who is taking insulin to treat


diabetes, which of the following drugs is to be used with extra
caution and advice to the patient?
A. Aspirin
B. Verapamil
C. Propranolol
D. Diltiazem
E. Nitroglycerin

294. Beta-blockers are used in all cases except:


A. Variant angina
B. Mild hypertension
C. Glaucoma
D. Thyrotoxicosis
E. Atrial arrhythmias

295. Which of the following drugs can relieve angina pectoris by


decreasing myocardial work, but may precipitate congestive heart
failure?
A. Phentolamine
B. Sthrophanthine
C. Propranolol
D. Atropine
E. Enalapril
296. The advantages of metoprolol over propranolol include all of the
following except
A. More likely to be effective in sinus tachycardia
B. Less likely to provoke bronchoconstriction
C. More likely to be effective in sinus bradycardia
D. Less likely to produce temporary rise in peripheral resistance at
the start of therapy
E. Less likely to produce sudden rises in BP after physical exertion

297. All the following statements are true about Enalapril except
A. A prodrug
B. Used to treat hypertension
C. Used to treat CHF
D. Inhibits ACE
E. Blocks Angiotensin-1

298. Calcium channel blocker with maximum peripheral action is:


A. Verapamil
B. Nifedipine
C. Diltiazem
D. Papaverine
E. Molsidomine

299. In a patient with myocardial infarction, if congestive cardiac


failure develops, the dose of digoxin should be:
A. Reduced
B. Increased
C. Unaltered
D. None of the above

300. Drugs that may precipitate variant angina pectoris are:


A. Beta blockers
B. Nitrites
C. Alpha blockers
D. Calcium channel blockers
E. Benzodiazepines

301. After tooth extraction the patient experienced persistence chest


pain. In result of sublingual use of an antianginal agent the chest
pain disappeared, but patient complained on headache and dizziness.
What agent did the patient use?
A Anaprilin (Propranolol)
B. Validolum
C. Nitroglycerine (Glyceril trinitrate)
D. Verapamil
E. Metoprolol

302. A patient had been suffering from stenocardia accompanied by


arterial hypotension. Indicate the anti anginal drug, which does not
cause decrease of blood pressure.
A Pentoxyphilline
B. Anaplilinum
C. Nitroglycerinum
D. Amiodaronum
E. Nifedipin

303. A 27 y.o. man with angina pectoris was admitted to a cardiologic


department. A phosphodiesterase inhibitor was included to the complex
therapy of the patient. Concentration of what substance will increase
in the cardiac muscle?
A ATP
B. Cyclic-AMP
C. ADP
D. GMP
E.AMP

304. A 32-years-old patient after intensive psychoemotional stress


suddenly experienced acute pain in heart area irradiating into left
arm. Nitroglycerin abolished the attack after 10 min. What pathogenic
mechanism is the leading reason in pain development in this case?
A Spasm of coronary vessels
B. Increasing of oxygen demand of myocardium
C. Dilation of peripheral blood vessels
D. Compression of coronary vessels
E. Obstruction of coronary vessels

305. To the patient, suffering from stenocardia attacks,


administration of long-acing nitrates is indicated. Find out this drug
from listed.
A. Nitroglycerin
B. Isosorbide mononitrate
C. Sodium-nitroprusside
D. Verapamil
E. Phenihydine (Nifedipine)

306. A 48 y.o. patient with the diagnosis of ischaemic heart disease


is getting medical treatment. Specify the antianginal agent that has
benefits to improve endocardial circulation:
A. Anaprilinum (Propranolol)
B. Nitroglycerin
C. Prazosin
D. Papaverine
E. Euphylline (Theophylline)

307. A patient who had been treated with a vitamin agent for the
prophylaxis of brain vessel constriction complained of unpleasant
sensations: blushing of upper part of the body, vertigo, flushing of
blood to the head. What agent exerts this effect?
A. Nicotinic acid
B. Thiamini bromide
C. Riboflavin
D. Tocopherol acetate
E. Anaprilinum (Propranolol)

308. A 3 y.o. child is delivered to an admission department with signs


of the nitrates poisoning such as cyanosis, dyspnoea, cramps. What is
the pathogenetic mechanism that underlines these signs?
A. Formation of a methemoglobin
B. Formation of an oxyhemoglobin
C. Formation of a carbhemoglobin
D. Formation of carboxyhemoglobin
E. Formation of the reduced hemoglobin

309. A 55- year-old man comes in for a check-up and complains that he
is having some difficulty in «starting to urinate». Physical
examination indicates that the man has a BP of 160/100 mm Hg and a
slightly enlarged prostate. Which of the following medications would
be useful in treating both of these conditions?
A. Doxazosin
B. Labetalol
C. Enalapril
D. Phentolamine
E. Propranolol

310. Which of the following drugs is a Ca2+ blocker selective on


cerebral blood vessels?
A. Nifedipine
B. Amlodipine
C. Nimodipine
D. Diltiazem
E. Verapamil
311. In a patient with hypertension at regular use of an
antihypertensine agent, cough has appeard. What agent can cause the
given adverse effect?
A. Prazosin
B. Verapamil
C. Clophelinum (Clonidinum)
D. Dichlothiasidum
E. Enalapril

312. One of the following signs is not a side effect of Captopril:


A. Cough
B. Renal dysfunction
C. Hyperkalaemia
D. Hemolytic anemia
E. Fever

313. All of the following drugs produce a significant decrease in


peripheral resistance except:
A. Chronic administration of diuretics
B. Hydralazine
C.beta-blockers
D. ACE inhibitors
E. Clofeline

314. Which of the folowing drugs should not be given to a pregnant,


hypertensive women?
A. Hydrochlorothiazide
B. Propranolol
C. Aminophylline
D. Enalapril
E. Verapamil

315. A 54-year-old man suffering from hypertension complains of severe


tachycardia after administration of antihypertensive drug. Which of
the following antihypertensives is most likely to cause reflex
tachycardia?
A. Prazosin
B. Verapamile
C. Hydralazine
D. Propranolol
E. Captopril

316. A 45 year-old hypertensive male has manifested hypoglycemia after


administration of antihypertensive drug. From the list of
antihypertensive drugs below select the one most likely to lower
plasma sugar.
A. Propranolol
B. Prazosine
C. Nifedipine
D. Captopril
E. Verapamil

317. A 17-y.o. man presents the symptoms of the familial


hypercholesterolemia. He was prescribed an agent inhibiting enzyme 3-
hydroxy-3-methylglutaryl-CoA reductase. What was the agent?
A. Simvastatine
B. Nicotinic acid
C. Gemfibrozil
D. Cholestiramine
E. Probucol

318. A patient 55 y.o. has suffered from the arterial hypertension for
10 years. Once in the evening he felt fatigue, acute headache,
darkening in the eyes, nausea. The patient was urgently delivered to a
hospital. The clinical examination revealed the increase of BP
(210/110 mm Hg). The diagnosis «hypertensive crisis» was made.
Administer the agents to treat hepertensive crisis.
A. Anapriline and Captopril
B. Clofeline and Furosemide
C. Captopril and Verospirone
D. Atenolol and Verospirone
E. Anapriline and Amiloride

319. A patient suffered from the arterial hypertension for a few


years. She had some accompanying diseases: chronic bronchitis with
asthmatic component, chronic gastritis with increased acid-producing
function. Indicate the antihypertensive drug, which is
contraindicated:
A. Propranolol
B. Captopril
C. Reserpine
D. Octadine
E. All above

320. Drug of choice in hypertensive crisis of pheochromocytoma is:


A. Clofeline
B. Capoten
C. Amlodipine
D. Phentolamine
E. Drotaverine

321. A 40 y.o. patient complains of intensive heartbeats, sweating,


nausea, vision impairment, arm tremor, hypertension. From his
anamnesis: 2 years ago he was diagnosed with pheochromocytoma. What
hypotensive agent should be administered?
A. Captopril
B. Prazosine
C. Drotaverine
D. Nifedipine
E. Platiohylline

322. A patient with II stage hypertension has been taking one of


hypotensive medications. After a time AP decreased, but the patient
started complaining of flaccidity, sleepiness, indifference. A bit
later he felt stomach pain. He was diagnosed with ulcer. What
hypotensive medication has the patient been taking?
A. Furosemide
B. Verapamil
C. Reserpine
D. Dibazole
E. Captopril

323. A patient had been suffering from hypertonic disease had been
treated for a long time with the drug from the group of Rauwolfia
alkaloids and began to complain of heartburn, pain in the epigastrial
area and bad mood. Indicate the drug which caused these complications:
A. Octadinum
B. Reserpinum
C. Clophelinum
D. Prasosine
E. Enalapril

324. A patient who had been suffering from arterial hypotension had
taken a hypotensive drug, but in an hour his AP increased and 2 hours
after it decreased. Indicate this antihypertensive agent:
A. Nifedipinum
B. Prazosinum
C. Captopril
D. Anaprilinum
E. Octadinum

325. A patient had been suffering from hypertonic disease accompanied


by chronic bronchitis with asthmatical component. Indicate the drug
which is contraindicated due to it‘s action on the bronchi:
A. Anaprilinum
B. Captopril
C. Prazosinum
D. Nifedipin
E. Dichlothiazidum

326. A doctor has administered clonidine (clophelinum) to a patient to


combat hypertensive crisis. What group does the named agent belong to?
A. Agonists of central alfa2 adrenoceptors
B. Antagonists of central alfa2 adrenoceptors
C. Agonists of central L1 –imidazoline receptors
D. Antagonists of central L1 –imidazoline receptors
E. Non-selective agonists of central alfa2-
adrenoceptors

327. A doctor has administered clonidine (clophelinum) to a patient to


treat hypertensive crisis. What class of antihypertensive drugs does
the named agent belong to?
A. Drugs affecting the renin-angoitensin system
B. Peripheral neurotropic
C. Diuretics
D. Central neurotropic
E. Myotropic (vasotropic) hypotensive agents

328. A woman addressed to the doctor complaining of headache, fatigue,


insomnia. Examination revealed high level of AP (200/100 mm Hg). A
doctor administered a highly effective drug the mechanism of action of
which is the inhibition of the anginomotor center activity due to
excitation of a-adrenoreceptors and I-1-imidazooline receptors of NTS
zone of medulla oblongata. Indicate this drug:
A. Phentrolamine
B. Reserpine
C. Hydrochlorthiazide
D. Clophelinum (clonidine)
E. Captopril

329. It was observed primary shorterm increase of AP in a patient


after IV administration of a hypertensive agent. Indicate this
preparation:
A. Clophelinum (clonidine)
B. Reserpine
C. Prazosine
D. Anaprilin (propranololo)
E. Hydrochlorthiazide (dichlothiazidum)
330. Stable arterial hypertension has developed in a patient suffering
from chronic glomerulonephritis. Indicate the most effective group of
drugs to treat this patient:
A. Ganglioblockers
B. Myotropic spasmolitics
C. alfa-adrenoblockers
D. beta-adrenoblockers
E. Angiotensine converting enzyme inhibitors

331. A patient suffering bronchial asthma had been taking tablets


which caused insomnia, headache, increased BP. what medicine can cause
such complication?
A. Chromolin sodium
B. Ephedrine
C. Euphylline
D. Adrenaline
E. Isadrine

332. During a surgical operation on the patient in deep general


anesthesia a danger of apnea was developed. To prevent this
complication 1 ml of cytitonum was introduced to the patient
intravenously, but the beneficial effect wasn‘t reached.
Indicate the analeptic which should be used in this situation:
A. Cordiamine
B. Coffeine
C. Bemegridum
D. Sulphocamphocaine
E. Corasol

333. An adult patient with COPD is being managed with several drugs,
one of which is inhaled Ipratropium bromide. What is the main effect
that accounts for the beneficial effects of this drug?
A. Blockade of endogenous bronchoconstrictor mediator
B. Enhanced release of epinephrine from the adrenal medulla
C. Inhibition of cAMP breakdown via phosphodiesterase inhibition
D. Stimulation of adenylyl cyclase
E. Prevention of antigen-antibody reactions that lead to mast cell
mediator release

334. A young boy with asthma receives Montelukast. What is the main
mechanism by which this drug occurs its beneficial effects?
A. Prevention of antigen-antibody reactions that lead to mast cell
mediator release
B. Enhanced release of epinephrine from the adrenal medulla
C. Stimulation of adenylyl cyclase
D. Blockade of receptors for pro-inflammatory and
bronchoconstrictor mediators
E. Inhibition of cAMP breakdown via phosphodiesterase inhibition

335. The most potent bronchoconstrictor is:


A. Histamine
B.Bradykinin
C.Leukotriens
D. Acetylcholine
E. Adenosine

336. A 12 y.o. boy with history of asthma complained of cough, dyspnea


and wheezing. His symptoms became so severe that his parents brought
him to the emergency room. Physical examination revealed diaphoresis,
dyspnea, tachycardia and tachypnea. His respiratory rate was 42/min,
pulse rate was 110/ min and BP was 130/70 mm Hg. Choose the most
appropriate drug to reverse the bronchoconstriction rapidly:
A. Methylprednisolone
B. Salmoterol
C. Cromolyn
D. Ipratropium
E. Salbutamol

337. A 12 y.o. boy with history of asthma complained of cough, dyspnea


and wheezing. His symptoms became so severe that his parents brought
him to the emergency room. Physical examination revealed diaphoresis,
dyspnea, tachycardia and tachypnea. His respiratory rate was 42/min,
pulse rate was 110/ min and BP was 130/70 mm Hg. Choose the drug most
likely to provide sustained resolution of the patients symptoms?
A. Methylprednisolone
B. Cronomolyn
C. Salmoterol
D. Ipratropium
E. Salbutamol

338. A 35-year-old woman was admitted to the surgical department with


the symptoms of acute pancreatitis: vomiting,diarrhea,bradycardia,
hypotension, weakness, dehydration of the organism, amylase of the
blood plasma – 128U. What agent should be used first of all?
A. Ephedrine
B. Platyphylline
C. Contrycal (Aprotinin)
D. Drotavreine (No-spa)
E. Naloxone
339. Indicate the mechanism of the emetic action of apomorphine:
A. Stimulation of the parasympathetic ganglions
B. Activation of chemoceptors of trigger zone of the
emetic center in medulla oblongata
C. Irritation of the gastric mucus
D. Increase of the tonicity of smooth muscles
E. Increase of secretion of hydrochloric acid

340. A doctor has administered Metoclopramide to a 37 y.o. man with


the symptoms of acute pancreatitis and severe vomitus. Indicate the
mechanism of action of this drug.
A. Blocks M-cholinoceptors
B. Blocks dopamine D2-receptors
C Stimulates M-cholinoceptors
D. Inhibits dopamine D2-receptors
341. Hypothalamic ventromedian nuclei stroke was diagnosed in a 50
y.o. woman. Her behavior is characterized by unsatisfied hunger,weight
gain. Which drug may be used to reduce her appetite?
A. Etimizol
B.Mazindol
C. Piracetam
D. Lipostabil
E. Caffeine

342. Methotrexate (structural analogue of the folic acid, which is


competitive inhibitor of the dihydrofolatreductase) is prescribed for
treatment of the malignant tumor.
On which level does Methotrexate hinder synthesis of the nucleic
acids?
A. Replication
B. Reparation
C. Processing
D. Transcription
E. Mononucleotide synthesis

343. A patient with the ulcer of the stomach with the increased acid-
producing function was administered an agent capable to inhibit
essentially 100% of gastric acid secretion in a single daily dose.
What agent has been used?
A. Atropine
B. Ranitidine
C. Famotidine
D. Omeprazole
E. Platiphylline
344. A patient with a severe arthritis will be placed on long-term
therapy with Indomethacin. Which of the following drugs is the most
apropriate choice to administer as an add-on (adjunct) to prevent
gastric ulcers caused by this NSAID?
A. Misoprostol
B. Celecoxib
C. Nimesulide
D. Cimetidine
E. Meloxicam

345. A liquidator of a breakdown at a nuclear power plant who was


irradiated complained about vomiting that occurs all of a sudden. What
medication should be prescribed?
A. Metoclopyramide
B. Reserpine
C. De-Nol
D. Atropine
E. Aeron

346. The alternative usage of Dichlotiazide, Etacrinic acid and Lasix


did not cause a marked diuretic effect in the patient with developed
peripheral edema. The aldosterone level in the plasma is increased.
Indicate which medicine should be prescribed:
A. Amilorid
B. Spironolactone
C. Mannit
D. Clopamid
E. Urea

347. A doctor administered Allopurinol to a 37-year old man with the


symptoms of gout. What pharmacological action of Allopurinol ensures
therapeutical effect?
A. By increasing Uric Acid excretion
B. By inhibiting leukocyte migration into joint
C. By inhibiting Uric Acid synthesis
D. By increasing Uric Acid biotransformation
E. By general anti-inflammatory effect

348. Which of the following enzymes is responsible for the ultimate


formation of uric acid, which contributes to hyperuricemia and the
pathophysiology of chronic and acute gout?
A. 5‘-Lipoxygenase
B. Xanthine oxidase
C. Phospholipase
D. Cyclooxygenase-1
E. Cyclooxygenase-2
349. Which of the following diuretics promote Ca2+ reabsorption?
A.Verospirone
B.Lasix
C.Amiloride
D.Indapamide
E.Hydrochlorothiazide

350. There is a pregnant woman in a delivery room that on term of


gestation should already have labors. The uterus cervix is not opened
at inspection. Indicate an agent for acceleration of uterus cervix
ripening and for stimulation of parturition activity:
A. Progesterone
B. Prostin E2 (Dinoprostone)
C. Estrone
D. Testosterone
E. Ergometrine

351. In connection with disturbances of contractility of myometrium in


postpartum period, the parturient woman had uterine bleeding. What
drug should administer in this situation?
A. Sodium oxybutyrate
B. Ergotamine
C. Salbutamol
D. Dinoprost
E. Atropine

352. Examination of a patient revealed hyperkaliemia and hyponatremia.


What diuretic was administred?
A. Verospirone
B. Furosemide
C. Hydrochlorthiazide
D. Bumetanide
E. Ethacrinic acid

353. A patient with chronic cardiac insufficiency has been treated


with cardiotonic drugs and a thiazide diuretic, but in spite of it
there are still edemata and risk of ascites. What medication should be
prescribed to amplify diuretic effect of the applied drugs?
A. Amiliride
B. Furosemide
C. Manitole
D. Spironolactone
E. Clopamide
354. A 35 y.o. patient who often consumes alcohol was treated with
diuretics. There appeared serious muscle and heart weakness, vomiting,
diarrhea. AP – 100/60 mm Hg, depression. This condition is caused by
intensified excretion with of:
A. Calcium
B. Potassium
C. Chlorine
D. Phosphates
E. Sodium

355. To a pregnant woman with a history of habitual abortion, a


Vitamin containing therapy was given. Select out vitamin that promotes
pregnancy preservation:
A. Tocopherol
B. Pyridoxal phosphate
C. Folic acid
D. Rutin
E. Cyanocobalamine

356. A 34 y.o. woman has arrived to the obstetrician department


because of threatened (habitual) abortion. What tocolytic agent is
required to administer to the patient?
A. Folliculin
B. Fenoterol (Partusisten)
C. Ethinylestradiol
D. Gonadotropin menopause
E. Estradiol

357. There is an inhibited coagulation in the patient with the bile


ducts obstruction, bleeding due to the low level of absorption of a
vitamin. What vitamin is in deficiency?
A. A
B. D
C. E
D. K
E. Carotene

358. A 2 y.o. child has got intestinal dysbacteriosis, which results


in hemorrhagic syndrome. What is the most probable cause of the
hemorrhage in the child?
A. Vitamin K deficiency
B. PP hypovitaminosis
C. Activation of tissue tromboplastin
D. Hypocalcaemia
E. Fibrinogen defiiciency
359. Inhibition of leukopoeisis is observed in a 43 y.o.
roentgenologist. The amount of leukocytes – 3,5x109/1. Specify the
agent to be used for correction of leukopoisis.
A. Ferroplexum
B. Hemostimulinum
C. Cyanocobalaminum
D. Ascorbic acid
E. Pentoxylum

360. Antidote for heparin is:


A. Protamine sulfate
B. EDTA
C. Vicasole
D. Deferoxamine
E. Ascorbinic acid

361. Indicate the mechanism of the heparin action:


A. Inhibition of synthesis of the clotting factors in the liver
B. Inhibition of thrombin
C. Inhibition of the conversion of prothrombin to
thrombin
D. Activation of profibrinolysin
E. Causes proteolysis of plasminogen molecules to plasmin

362. Indicate the clinical use of cyanocobalamine:


A. Leucopoenia
B. Malignant megaloblastic anemia
C. Acute leucosis
D. Gout
E. Thyrotoxicosis

363. Indicate the mechanism of antiplatelet (antithrombotic action) of


aspirin:
A. Inhibition of PDE that promotes the accumulation of cAMP
B. Stimulation of adenilat cyclase in thrombocytes
C. Increase of prostacyclin synthesis
D. Block of adenosine uptake
E. Inhibition of cyclooxygenase and prevention of
synthesis of thromboxane

364. Indicate the drug used locally for the arrest of hemorrhages from
the small vessels.
A. Calcium chloride
B. Contrical
C. Amben
D. Thrombinum
E. Vikasol

365. Indicate the mechanism of the streptokinase action:


A. Inhibition of synthesis of the clotting factors in the liver
B. Inhibition of thrombin
C. Inhibition of the conversion of prothrombin to thrombin
D. Activation of profibrinolysin
E. Causes proteolysis of fibrin

366. A patient 65 years, was delivered to the hospital in connection


with thromboembolism of pulmonary artery, which developed after
physical exertion and required intense therapy. Choose the drug of
choice for this patient:
A. Aspirin
B. Dipiridamole
C. Ambciximab
D. Streptokinase
E. Clopidogrel

367. The anticoagulant activity of Neodicumarin can be potentiated by


all of the following drugs EXCEPT:
A. Rifampicin.
B Aspirin
C Pentoxifylline
D Cimetidine
E Disulfiram

368. Examination of a 43 y.o. anephric patient revealed anemia


symptoms. What is agent should be prescribed?
A. Epoetinum (Epogen)
B. Vitamin B12
C. Vicaslum
D. Folic acid
E. Fercovenum

369. A 27 y.o. woman suffers from hypochromic anemia in connection


with significant blood loss after thyroidectomy. What drug should be
administered to the patient?
A. Cyanocobolamine (Vitamin B12)
B. Coamid
C. Folic acid
D. Pentoxil
E. Ferrum-lek
370. During a competition indirect proofs of utilization of the
forbidden drug erythropoietin are found in the sportsmen-skiers by
results of the dope-control. Define, what is not typical for
erythropoietin?
A. It is preoduced by recombinant technologies
B. It decreases level of erythrocytes in blood
C. It is synthesized in kidney at conditions of hypoxia
D. It stimulates erythropoiesis
E. It can be used for treatment of anemia on a background of chronic
renal failure

371. To the patient suffering from stenocardia and taking isosorbide


mononitrate, has been given an agent with antiaggregative activity.
Indicate this agent.
A. Anaprilin (Propranolol)
B. Acetylsalicylic acid
C. Nifedipine
D. Nitroglycerin
E. Validol

372. The patient with myocardial infarction had been receiving each 6
hours an anticoagulant IV. In a few days, he has developed bleeding
gums, nasal bleeding; erythrocytes in urine have appeared. What drug
was introduced to the patient?
A. Aminocapronic acid
B. Neodicumarin
C.Vicasol
D. Calcium gluconate
E. Heparine

373. Symptoms of internal bleeding developed at a patient of


gynecologic department. What agent should be given for inhibition of
fibrinolysis and arrest of hemorrhage?
A. Calcium chloride
B. Dicinone
C. Vicasol
D. Contrical
E. Fibrinogen

374. A 38 y.o. woman has been on a slim diet consisted of 2 cups of


coffee without sugar, 3 crackers from white bread, 2 eggs or 2
sausages a day for a month. She lost 5 kg her weight but developed
headaches, often nasal bleedings, bleeding sickness of gums, shaking
of teeth, shelling of skin and shedding of hair. What vitamin drug is
expedient in this case?
A. Ascorbinic acid
B. Cyanocobalaminum
C. Folic acid
D. Vicasolum (Menadione)
E. Retinol acetate

375. A 32 y.o. woman visited gynecologist with the complaints of the


usual abortions during the first months of pregnancy and the disorders
of the menstrual cycle. Examination revealed the signs of the sexual
infantility and impairment of ovulation in the ovaries. A doctor
suggested the diagnosis of pituitary insufficiency. Administer the
hormonal drug to treat this patient:
A. Corticotrophin
B. Glibenclamide
C. Oxitocin
D. Gonadotropin menopausticum
E. Pituitrin

376. A patient had been suffering from atopic bronchial asthma for a
long time. Last time he was treated with Prednisolon because of
worsening of his state. Long treatment with Prednisolon caused the
development of adverse effects: edemas, increase of glucose
concentration in the blood, increased excitability and sleeplessness.
Indicate hormonal drug, which should be administered for prevention of
this complication:
A. Androcur (Cyproterone acetate)
B. Retabolile
C. Glibenclamide
D. Tocoferol acetas
E. Corticotropine

377. Determine the drug of choice for hyperprolactinemia:


A. Bromocryptine
B. L-Dopa
C. Oxitocin
D. PItuitrin
E. Estradiol

378. Glucocorticoids are contraindicated in all cases except:


A. Organ transplants
B. Peptic ulcers
C.Hypertension
D. Diabetes Mellitus
E. Tuberculosis
379. The corticosteroid without any glucocorticoid activity is:
A. Aldosterone
B. Fludrocortisone
C. DOCA (desoxycorticosteron acetas)
D. Cortisol
E. Triampicinolone

380. The following states are adverse reactions of oral contraceptives


except:
A. Tromboembolism
B. Weight gain
C.Carbohydrate intolerance
D. Galactorrhea
E. Hyperlipidemia

381. A woman who has been taking an oral contraceptive (estrogen +


progestin) for 3 years is diagnosed epilepsy and started on Phenytoin.
Which of the following states is the most probable consequence of
adding Phenytoin?
A. Reduced contraceptive efficacy
B. Thromboembolism from the estrogen component of the contraceptive
C. Increased risk of craniofacial abnormalities in the fetus
D. Aplastic anemia, requiring stopping both drugs immediately
E. Seizures from increased Phenytoin clearance

382. A 32 y.o. man has complained of a headache, fatigue, thirst,


pains in the spine and joints for the last 2 years. Clinically it was
observed disproportional enlargement of hands, feet, nose,
superciliary arches. He noted that he needed to buy 3 times bigger
shoes. What is the main reason of such disproportional enlargement of
different parts of the body?
A. Increased sensitivity of the tissues to growth hormone
B. Increased sensitivity of the tissues to insulin
C. Cartilaginous tissue proliferation under growth
hormone influence
D. Joint dystrophy development
E. Joints chronic inflammation development

383. A patient has been taking glucocorticoids for a long time. When
the preparation was withdrawn he developed the symptoms of disease
aggravation, decreased AP and weakness. What is the reason of this
condition?
A. Sensibilization
B. Appearance of adrenal insufficiency
C. Cumulation
D. Hyperproduction of ACTH
E. Idiosyncrasy

384. A 2-year-old child experienced convulsions because of lowering


Ca2+ions concentration in the blood plasma. Which treatment will be
the most useful?
A. Parathyroid hormone
B. Insulin
C. Calcitrine
D. Calcium chloride
E. Parathyroid hormone + calcium chloride

385. A 17 y.o. female suffers from tachycardia at rest, weight loss,


excessive sweating, exophtalmos, and irritability, elevated Thyroxine
in her serum. What treatment should be provided to the patient?
A. Thyrotropin
B. Calcitonin
C. Reserpine
D. Mercazolil (Methimazole)
E. Prednisolone

386. Many therapeutic insulins are often modifications of ‗regular‘


insulin. The modifications include substituting some amino acids in
the protein using recombinant DNA technology, conjugating insulin with
NPA (neutral protamine Hagedorn), or combining it with zinc. For all
these insulins, which of the following statement is the one common
result of such changes?
A. Enablingadministration by either SC or IV routes
B. Prevention of cellular K+ uptake
C. Stimulation of pancreatic insulin synthesis
D. Modification of onsets and durations of action
E. Elimination of allergic responses

387. Taking oral contraceptives containing sex hormones inhibits


endogenous pituitary secretion. From the list of hormones below select
the one when secretion is inhibited while taking oral contraceptives
containing sex hormones.
A. Follicle-stimulating hormone
B. Oxitocine
C. Pituitarine
D. Growth hormone
E. Thyroid-stimulating hormone

388. At a 56 y.o. patient that complaints on thirst and frequent


urination it was established diagnosis of diabetes mellitus and was
given an agent – Butamide (Tolbutamide). What is the mechanism of
action of this agent?
A. Promotes glucose utilization by peripheral tissues
B. Stimulates beta-cells islets of Langerhans
C. Suppresses beta-cells isles of Langerhans
D. Facilitate glucose transport through cellular membranes
E. Inhibits glucose absorption in the intestine

389. Diagnosis of diabetes mellitus was established to a 68 y.o.


patient that complains on thirst and frequent urination and was
administered an agent – Metformin. What is the mechanism of action of
this agent?
A. Promotes glucose utilization by peripheral tissues
B. Stimulates beta-cells isles of Langerhans
C. Suppresses beta-cells isles of Langerhans
D. Inhibits glucose transport through cellular membranes
E. Inhibits glucose absorption in the intestine

390. Testosterone and its analogs increase volume of skeletal muscles


that make this drug useful for the treatment of myodystrophy.
Interaction with what cellular substrate determines this action?
A. Membrane receptors
B. Ribosomes
C. Nuclear receptors
D. Chromatin
E. Protein-activator of transcription

391. A 35 y.o. woman that has been treated for a long time because of
chronic polyarthritis is complaining on rising of AP, redistribution
of fat tissues, disturbances of menstrual cycle. With what drug
administrations these complain are related to?
A. Beclomethasone
B. Phenylbutazone (Butadion)
C. Meloxicam
D. Prednisolon
E. Diclofenac-natrium

392 The 32 y.o. woman was taking a contraceptive Tri-Regol for a long
time. What does not belong to the adverse effects of this drug?
A. Thrombosis
B. Pyelonephritis
C. Weight gain
D. Hepatotoxicity
E. Osteoporosis
393. For the prevention of undesirable pregnancy the doctor–
gynecologist has recommended to the 24 y.o. woman a peroral
contraceptive drug, which should be applied immediately after the
sexual intercourse. Specify this drug.
A. Rigevidon
B. Triziston
C. Levonorgestrel (Postinor)
D. Estradiol
E. Proginova

394. A 27 y.o. woman who from time to time uses peroral contraceptives
has addressed to the doctor with a question on composition of these
agents. What steroid substances are included to postcoital peroral
contraceptives?
A. Gonadotropins
B. Progestins
C. Androgens
D. Anabolic steroids
E. Glucocorticoids

395. A 27 y.o. woman has arrived to the obstetrician department


because of threatened (habitual) abortion. What hormonal agent is
required to administer to the patient?
A. Folliculin
B. Progesterone
C. Ethinylestradiol
D. Gonadotropin menopause
E. Estradiol

396. Many kinds of pathology (inflammation, pulmonary edema, shock of


different origin) are accompanied by increasing of vascular
permeability. What agent from the named drugs can be used for
counteraction to this at any of the named kinds of pathology?
A. Dimedrolum (Diphenhydramine)
B. Acid acetylsalicylic
C. Indomethacin
D. Prednisolone
E. Beclomethasone

397. Continious taking of a drug can result in osteoporosis, erosion


of stomach mucous membrane, hypokaliemia, retention of sodium and
water, reduced content of corticotropin in blood. Name this drug:
A. Digoxin
B. Hydrochlorothiazide
C. Reserpine
D. Indometacin
E. Prednisolone
398. A patient with infectious mononucleosis has been taking
glucocorticoids for two weeks. He was brought into remission, but he
fell ill with acute attack of chronic tonsillitis. What action of
glucocorticoids caused this complication?
A. Immunosuppressive
B. Anti-inflammatory
C. Antitoxic
D. Antishock
E. Antiallergic

399. A 3 y.o. child has convulsions as s result of lowered


concentration of Ca2+ in blood plasma. What hormone should be
administered concomitantly with calcium agent?
A. Calcitrin
B. Parathyroid hormone (Parathormone)
C. Miacalcic
D. L - Tyroxine
E. Prednisolone

400. A 45 y.o. woman suffers from Cushing‘s syndrome – steroid


diabetes. Biochemical examination revealed: hyperglycemia,
hypochloremia. Which of the undermentioned processes is the first to
be activated? 2007
A. Glycogenolesis
B. Glycolysis
C. Gluconeogenesis
D. Glucose reabsorption
E. Glucose transport to the cell

401. Examination of a 60 y.o. patient revealed hyperglycemia and


glucosuria. A doctor administered him a medication for internal use.
What medication is it?
A. Oxytocin
B. Glibenclamid
C. Furosemide
D. Pancreatine
E. Corglycon

402. A patient with collagenesis has been taking prednisolone for a


long time. Hypokaliemia development caused spastic pain of skeletal
muscles. What medication should be used in order to correct potassium
exchange?
A. Noshpa
B. Dithylinum
C. Panangin
D. Thyrocalcitonin
E. Diazepam

403. Parents of a 10 y.o. boy consulted a doctor about extension of


hair-covering, growth of beard and moustache, low voice. Intensified
secretion of which hormone must be assumed?
A. Cortisol
B. Oestrogen
C. Somatotropin
D. Testosterone
E. Progesterone

404. Arachidonic acid is metabolized by two main pathways:


cyclooxygenase and lipooxygenase. The latter, initially involving 5‘-
lipooxygenase, is responsible for the synthesis of one of the
following substances. Choose the correct answer.
A. Leukotriens
B. Prostacyclin
C. Prostaglandins
D. Thromboxanes
E. Platelet-activating factor

405. A 23 y.o. man suffers from allergic seasonal coryza caused by the
ambrosia blossoming. What medicine stabilizing adipose cells can be
used for prevention of this disease?
A. Phencarol
B. Diazoline
C. Dimedrol
D. Tavegyl
E. Ketotifen

406. A woman with an allergic disorder experiences significant


bronchoconstriction and urticaria, and histamine is a main mediator in
these responses. Which of the following drugs may pose extra risk for
this patient – not because it has any bronchoconstrictor effects, but
because it quite effectively releases histamine from mast cells?
A. Atropine
B. Neostigmine
C. Propranolol
D. Pancuronium
E. d-Tubocurarine

407. A 30 y.o. woman with a 3-year history of chronic renal failure


requiring dialysis consents to transplantation. A donor kidney becomes
available. She is given Cyclosporine to prevent rejection. Which of
the following states is the most probable adverse effect of the drug?
A. GIT ulceration
B. Seizures
C. Hepatotoxicity
D. Nephrotoxicity
E. Bone marrow depression

408. Which of the following drugs is non-sedative H1 histamine


receptor antagonist?
A. Diprazinum (Promethazine)
B. Dimedrolum (Diphenhydramine)
C. Suprastine
D. Loratadine
E. Tavegil (Clemastine)

409. A 22 y.o. woman has a runny nose and itching eyes from a bout
with the common cold. Dimedrolum (Diphenhydramine) provides
symptomatic relief. Indicate the mechanism of action that is most
likely associated this the named drug.
A. beta-adrenoceptors blockade
B.adrenergic stimulation
C. Histamine (H1) receptor blockade
D. MAO inhibition
E. Muscarinic receptor blockade

410. An asthma patient has symptoms flare-ups during hay fever


season.He visits the localsuperstore and buys an over-the-counter
antihistamine/allergy remedy containing Dimedrolum (Diphenhydramine).
After a few days of using it, his breathing becomes worse. You examine
him and conclude that what the patient viewed as the allergy cure was
actually the cause of the problems. Which of the following is the most
likely mechanism by which the Dimedrolum worsened this patient
condition?
A. Enhancing metabolic clearance of other asthma agents (lowering
their serum levels)
B. Drying the airways, increasing mucus viscosity
C. Releasing histamine
D. Causing bronchoconstriction by releasing more Ach in the airways
E. Blocking the endogenous bronchodilator effects of circulating
epinephrine

411. A 25 y.o. man has had a kidney transplant. He receives


Cyclosporine as part of the immunosupressant regimen. What is the main
mechanism of Cyclosporine‘s immunosupressant effects?
A. Inhibits calcinurenin and IL-2 synthesis that is
necessary for B and T cell proliferation
B. Inhibits antigen-activated lymphocytes, reduces responsiveness of
T-lymphocytes to IL-1, reduces IL-2 production by lymphocytes and
monocytes
C. Directly inhibits B and T lymphocyte proliferation
D. Directly destroys proliferating B and T lymphocyte proliferation
E. Blocks the CD3 site on T lymphocytes, blocks all T cell functions

412. Which of the following agents is the drug of choice for treating
anaphylaxis?
A. Epinephrine
B. Norepinephrine
C. Atropine sulfate
D. Dimedrolum (Diphenhydramine)
E. Theophylline

413. A 21 y.o. man with asthma has what is described as ‗aspirin


sensitivity‘ and experiences severe bronchospasm in response to even
small doses of the drug. The most likely explanation for this is that
the aspirin.
A. Induces hypersensitivity of H1 receptors on airway smooth muscles
B. Suppresses airway inflammatory processes
C. Blocks synthesis of endogenous prostaglandins that
have bronchodilator activity
E. Induces hypersensitivity of muscarinic receptor of airway smooth
muscles

414.A doctor administered a patient with allergic dermatitis a


H1_histamine blocker as a part of complex treatment. Name this
medication:
A. Hydrocortisone
B. Cromolin sodium
C. Prednisolone
D. Adrenaline
E. Loratadine

415. Dimedrolum (diphenhydramine) is administered to a patient with


urticaria to reduce itching rashes on the skin. What mechanism
provides its effeciency in this case?
A. Competitive blockade of H1-histamine receptors
B. Inhibition of synthesis of histamine
C. Suppression of release of histamine
D. Acceleration of histamine
E. Blockade of H2- histamine receptors
416. What drug should be administered to the patient who suffers from
rashes due to allergic reaction accompanied by reddening, edema, and
strong itch of skin, which causes sleeplessness?
A.Nitrazepamum
B. Dimedrolum (Diphenhydramine)
C. Chlorali hydrate
D. Natrii oxybutyrate (oxybate sodium)
E. Phernobarbitalum

417. The patient with allergic rhinitis has taken antihistamine drug
PO. In half an hour the patient felt dryness in mouth, retardation and
sleepiness. Indicate this drug.
A. Phernobarbitalum
B. Diazolinum (Mebhydrolin)
C. Dimedrolum (Diphenhydramine)
D. Diazepamum
E. Paracetamolum

418. A 40 y.o.parient is hospitalized with exprssed Quinke‘s edema


after stung of a bee. Indicate an antihistaminic drug to be introduced
without sedative and hypnotic action.
A. Suprastinum (Choloropyramine)
B. Phencarolum (Quifenadine)
C. Diazolinum (Mebhydroline)
D. Tavegilum (Clemastine)
E. Dimedrolum (Diphenhydramine)

419. A girl was treated with antibiotic from the group of


semisynthetic Penicillins due to acute bronchitis. On the 3 rd day of
treatment allergic dermatosis has developed. Indicate an antiallergic
drug which should be administered t the patient.
A. Suprastinum (Chloropyramine)
B. Levamisole
C. Aspirin (Acetylsalicylic acid)
D. Cyclosporin
E. Dexamethasone

420. A 21 y.o. patient has been diagnosed with allergic dermatitis.


The doctor has administered to him complex therapy including the
blocker of H1-histamine receptors. Indicate this drug.
A. Cromolin sodium (cromoglicic acid)
B. Diprazinum (promethazine)
C. Prednisolone
D. Adrenaline
E. Hydrocortisone
421. Indicate the group of antiallergic agents which Loratadine
belongs to:
A. Glucocorticoids
B. Membrane stabilizers
C. Antagonists of leucotriene receptors
D. Blockers of histamine receptors
E. Blockers of serotonine receptors

422. Treatment by anti-inflammatory drugs was administered to the


patient with rheumatic endocarditis. After a while hyperglycemia was
developed in him. What group of drugs is capable to provoke such side
effect?
A. Nonsteroid anti-inflammatory agents
B. d- Penicillamine
C. Methotrexat
D. Sulfasalazine
E. Glucocorticoids

423. The immunodepressive effect of Prednisolone is cause by:


A. Inhibition of protein synthesis due to activation of
gluconeogenesis
B. Inhibition of collagen synthesis
C. Activation of synthesis of inhibitors pf proteases
D. Inhibition of synthesis of mucopolysaccharides
E. Diminution of activity of plasmin

424. A 35 y.o. patient, suffering from bronchial asthma, is


hospitalized in a state of anaphylactic shock. What drug is necessary
to introduce first of all as first aid?
A.Dimedrolum (diphenhydramine)
B. Adrenaline
C. Chromoglicic-acid
D. Salbutamol
E. Ephedrine

425. Anaphylactic shock has developed at the patient after


intracutaneous test on sensitivity to Pencillin. The doctor had
administered a drug which eliminated bronchospasm and arterial
hypotension. Indicate this drug:
A. Mesatonum (Phenylephine)
B. Noradrenaline
C. Adrenaline
D. Atropine
E. Salbutamol

426. Many kinds of pathological states (inflammation, pulmonary edema,


shock of different origin) are accompanied by violation of
permeability of vessels. Which of the listed below drugs can be used
for elimination of the reaction at any of the termed states?
A. Aspirin (acetylsacylic-acid)
B. Indomethacinum
C. Dimedrolum (diphenhydramine)
D. Prednisolone
E. Beclomethasone

427. Indicate the drug which is the most expedient to use topically to
treat allergic dermatitis?
A.Sea buckthorn oil (Oleum Hippophae)
B. Solution of furacilinum (nitrofuran)
C. Liniment of methylsalicylate
D. Ointrent of inchyolum (ichthammol)
E. Hydrocortisone ointment

428. The doctor has administered a glucocorticoid Synaflanum


(fluocinoline) in the form of ointment to the patient with dermatitis.
What pharmacolodical effect is not characteristic for this drug?
A.Magnification of hyperemia
B. Diminution of itch
C. Diminution of edema
D. Deceleration of wound repair
E. Inhibition of skin immune response

429. Fluocinolone acetonide is administered to the patient, suffering


from allergic cheilitis for smearing of lipline and mucous of the
lips. Indicate the group which this drug belongs to?
A. Antragonists of histamine H1 receptors
B. Antragonists of histamine H2 receptors
C. Glucocorticoids
D.Membrane stabilizers
E. Antragonists of luekotriene receptors

430. A doctor administered Cromolyn sodium (cromoglycate sodium) to


the patient suffering from bronchial asthma to prevent attacks.
Indicate the principle of action of this drug:
A. Decrease of concentrations of immunoglobulins
B. Binding of free histamine
C. Stabilization of membranes of mast cells
D. Inactivation of histamine
E. Blockade of histamine receptors

431. A 45 y.o. woman suffers from allergic seasonal coryza caused by


Ambrosia blossoming. What drug from the group of stabilizers of mast
cells can be used for prevention of the disease?
A. Ketotifen
B. Phencarol
C. Tavegil
D. Dimedrol
E. Diazoline

432. Ketotifenum is administered to a patient with bronchial asthma.


Indicate mechanism of action of this antiallergic drug:
A. Blocks H2-histamine receptors
B. Blocks H1-and H2-histamine receptors
C. Inhibits synthesis of histamine
D. Blocks H1- histamine receptors
E. Activates enzymes which cleave histamine

433. What agent from the group of adrenomimetics is the most effective
for local use in allergic rhinitis?
A. Mesatonum (Phenylphine)
B. Naphthyzinum (Naphazoline)
C. Fenoterol
D. Salbutamol
E. Ephedrine

434. A 23 y.o. patient took Levomycetinum (chloramphenicole) for a


long time without doctor permission. The patient examination revealed
leucopenia. What drug should be administered for stimulation of
leucopoiesis?
A. Pentoxylum
B. Methotrexate
C. Mercaptopurine
D. Cyancobalamine
E. Prednisoline

435. The patient with chronic infectious disease requires treatment


with non specific immunostimulant agent. Indicate this drug.
A. Mercaptopurine
B. Methyluracil
C. Azathioprine
D. Actinomycin
E. Cyclophosphane
436. A 48 y.o. woman suffering from exacerbation of chronic pnenumonia
requires treatment with immunostimulant agent. Indicate this drug.
A. Sulfocamphocainum
B. Thymalinum
C. Biseptol (co-trimoxazole)
D. Dimedrolum (diphenhydramine)
E. Levamisole

437. Indexes of immune response are worsened in a patient during


chemotherapy of malignant tumour. What drug should be administred in
this state?
A. Thymalinum
B. Cyancobalamine
C. Iron preparations
D. Prednisolone
E. Acetylsalicylic acid

438. A drug from the group of immunostimulant, which is an analogue of


natural biogenic substrate and almost does not exert side effects is
administerered to a 4 y.o. child. Indicate this drug.
A. Thymalinum
B. Prodigiosanum
C. Interferon
D. Pyrogenalum
E. Levamisole

439. It is necessary to administer an antihistamine agent to a woman


suffering from seasonal vasomotor rhinitis, who works as a dispetcher
on the railway. Indicate the drug to treat the patient:
A. Cromolyn sodium (cromoglycate sodium)
B. Diprazinum (promethazine)
C. Prednisolone
D. Telfast
E. Hydrocortisone

440. A patient suffering from chronic generalized parodontitis


requires the treatment with immunostimulant agent, which also
possesses antihelmintic activity. Indicate this drug.
A. Pyrantel pamoate
B. Seeds of pumpkin
C. Cycloferon
D. Cyclosporine
E. Levamoisole
441. Which vitamin or nutrient, also an ingredient in some
prescription medications for severe, refractory acne vulgaris, is
―highly‖ teratogenic and should not be administered pregnant women?
A. Vitamin A
B. Vitamin B6
C. Vitamin C
D. Vitamin E
E. Vitamin K

442. Fat-soluble vitamins, compared with their water-soluble forms,


generally have a greater potential toxicity. They are:
A. Involved in more essential metabolic pathways
B. Involved in less essential metabolic pathways
C. Avidly stored by the body
D. Capable of dissolving membrane phospholipids
E. Administered in larger doses

443. The patient should be told to avoid taking supplemental Vitamin


B6 (pyridoxine) if he is being treated with one of these drugs:
A. Digoxin
B. Haloperidol
C. Niacin
D. Levodopa (Carbidopa)
E. Phenytoin

444. You have a patient who has been consuming extraordinarily large
amounts of alcohol for several years. He goes into acute withdrawal
and manifests nystagmus and bizarre ocular movements and confusion
(Wernicke‘s encephalopathy). Although this patient‘s alcohol
consumption pattern has been accompanied by poor nutrient intake
overall, you specifically manage the encephalopathy by administering
one of these drugs:
A. Vitamin A
B. Folic acid
C. Vitamin B12 (Cyanocobalamine)
D. Vitamin E (alfa-tocopherol)
E. Vitamin B1 (Thiamine)

445. Indicate the main feature of Trisaminum (Trometamol) that Sodium


bicarbonicum is unable:
A. It is available for taking orally
B. It is available for IV infusion
C. It penetrates into the cell, binding to hydrogen
ions outside and inside the cell
D. It does not penetrate into the cell that is why it eliminates
extracellular acidosis only
E. It increases BP

446. You are doing summer volunteer work at health clinic in a very
poor region of the world. A 19-y.o. woman is diagnosed Vitamin D-
resistant rickets. Apart from a high-dose of Vitamin D and oral
phosphate, an additional therapeutic approach might be used. What
agent should you choose?
A. Estrogen
B. Hydrochlorothiazide
C. Folic acid
D. Calcitriol
E. Vitamin B12

447. What drug from the list below is used to inhibit pancreas
function?
A. Urokinase
B. Pancreatin
C. Alteplase
D. Contrycal (Gordox)
E. Lydazum

448. The vitamin agent, deficiency of which can cause sterility in


experimental animals, and de applied to complex treatment of female
infertility. Specify this vitamin.
A.Vitamin A
B. Vitamin K
C.vitamin B1
D. Vitamin D
E. Vitamin E
449. A 45 y.o. patient was admitted to the hematologic department with
acute anemia: RBCs-1,5 x 10.12/L, Hb _80g%, colour index 1,3.
Hyperchromic anemia was diagnosed.Which drug should be administered
for treatment of this disease?
A. Hemostimuline
B. Ferroplexum
C. Cyanocobalamine
D. Ferrum-lek
E. Folics acid

450. Which of the acids below decreases permeability of connective


tissue structures, possesses antioxidant activity due to ability to be
transformed from the oxidized from into reduced and on the contrary?
A. Aspirin (acetylsalicylic acid)
B. Hydrochloric acid
C. Mefanamic acid
D. Ascorbic acid
E. Aminocaproic acid

451. For synthesis of the basic substances of connective tissue


(mucopolysaccharide and collagen) an essential agent is:
A. Nicotinic acid
B. Ascorbic acid
C. Folic acid
D. Salicylic acid
E. Acetylsalicylic acid

452. Radiation therapy is performed to the patient. What vitamin drug


with antioxidant properties is necessary to administer to increase
stability of tissues in this case?
A. Thiamine chloride
B. Vitamin B6
C. Ascorutinum
D. Cyanocobalamine
E. Folic acid

453. A dermatologist diagnosed red flat herpes in a 28 y.o. patient


and administered him under the locus of lesion 30 injections of a
vitamin drug, which participates in oxidative phoshorylation. Specify
this drug:
A. Nicotinic acid
B. Thiamine bromide
C. Ascorbic acid
D. Vitamin B6
E. Ergocalciferol

454. The patient who was treated by a vitaminic drug for prophylaxis
of vasospasms of the brain has developed complaints of the unpleasant
sensations related to taking of this medicine: reddening of the face
and the upper half of a trunk, giddiness, sense of flush of blood to a
head. For what drug the specified side effects are characteristics?
A. Tocofecol acetate
B. Nicotinamidum
C. Thiamine bromide
D. Nicotinic acid
E. Calcium pangamate
455. Diarrhea, dementia and dermatitis are observed at the patient.
What vitaminic drug should be included into a complex treatment?
A. Thiamin
B. Cyanobalamine
C. Nicotinamide (vitamin PP)
D. Calcium pantothenate
E. Cyanocobalamine

456. 55 y.o. woman suffers from hemaralopia (disturbance of vision in


darkness). What vitamin drug should be recommended her first of all?
A.Riboflavin
B. Tocoferol acetate
C. Pyridoxine
D. Ascorbic acid
E.Retinol acetate

457.Which vitamin promotes growth and development of epithelial cells,


including epidermal ones?
A. Retinol
B.Ergocalciferol
C. Ascorbic acid
D. Nicotinic acid
E. Lipoid acid

458. 37 years old patient suffers from hyperkeratosis, disturbance of


vision in darkness, frequent infectious deseases. What vitaminic drug
should be administered for treatment?
A. Pyridoxin
B. Retinol acetate
C. Riboflavin
D. Erdocalciferol
E. Tocoferol acetate

459. The patient had been taking vitamin D for a long time for
treatment of rickets. The phenomena of intoxication have developed:
depression of appetite, nausea, headache, fatigue, disorders of sleep,
increase of body temperature, changes in urine – hyaline cylinders,
protein, leucocytes. What vitaminic drug should be administered to
weaken arisen symptoms?
A. Vikasolum (Menadione)
B. Vitamin B12
C. Vitamin A (Retinol acetate)
D. Vitamin PP
E. Riboflavin
460. Parasthesia, xeroderma and sticking out fontanel are observed at
a 6 months child under the treatment by a vitaminic drug. Specify this
drug:
A. Retinol acetate
B. Pyridoxine
C. Riboflavin
D. Ergocalciferol
E. Tocoferol acetate

461. What of the listed below vitaminic drugs possesses expressed


radioprotective property?
A. Ergocalciferol
B. Thiamine chloride
C. Riboflavin
D. Tocoferol acetate
E. Folic acid

462. What enzymatic drug is used with the purpose of reduction of


density and rising of permeability of connective tissue structures?
A. Amylase
B. Lipase
C. Cocarboxylase
D. Cholinesterase
E. Lidase

463. According to clinical indications a patient was administered


pyridoxal phosphate. What processes is this medication intended to
correct?
A. Dissemination of purine nucleotide
B. Synthesis of purine and pyrimidine bases
C. Transamination and decarboxylation of aminoacids
D. Protein synthesis
E. Oxidative decarboxylation of kenotic acids

464. Examination of a man who hadn‘t been consuming fats but had been
getting enough carbohydrates dermatitis, poor wound healing, vision
impairment. What is probable cause of metabolic disorder?
A. Lack of oleic acid
B. Low caloric value of diet
C. Lack of palmitic acid
D. Lack of vitamins PP, H
E. Lack of linoleic acid, vitamins A, D, E, K
465. Removal of gall bladder of a patient has disturbed processes of
Ca2+ absorption through the intestinal wall. What vitamin will
stimulate this process?
A. PP
B. B12
C. C
D. K
E. D3

466. The patient with the diagnosis of focal tuberculosis of the


upper lobe of a right lung receives isoniazid in complex therapy.
After a while the patient started to complain on muscle weakness,
desensitization of the skin, disturbance of vision and coordination of
movement. What vitamin agent should be used for removing of these
phenomena?
A. Vitamin C
B. Vitamin B6
C. Vitamin E
D. Vitamin D
E. Vitamin B12

467. During hypertensive crisis magnesium sulfate has been introduced


to the patient and after that AP fell down. Which drug introduction
will remove this side effect of magnesium sulfate?
A. Sodium bromide
B. Potassium bromide
C. Trilon B
D. Ferrum-Lek
E. Calcium chloride

468. A patient with the abscess of the cut wound applied to the
traumatologist. The wound was washed with 3% Hydrogen Peroxide to be
cleaned from the pus. Foam was not observed. What caused inefficiency
of the drug?
A. Pus in the wound
B. Shallow wound
C. Inherited insufficiency of catalase
D. Low concentration of H2O2
E. Inherited insufficiency erythrocyte‘s phosphatdehydrogenase

469. A 23 y.o. patient addressed to an ophthalmologist with complaints


of eye discomfort, discharge of purulent exudate, disorders of vision.
Specify the antiseptic to rinse the eyes:
A. Lugol‘s solution
B. Potassium permanganate
C. Ammonium solution
D. Silver nitrate
E. Chlorhexidinum

470. For the preparation of a patient‘s burn skin surface a certain


medication was used. Its antiseptic action is provided by free oxygen
that segregates in presence of organic substances. Choose the right
answer:
A. Furacilin
B. Boric acid
C. Chlorhexidine
D. Potassium permanganate
E. Sodium bicarbonate

471. A 57 y.o. patient with varicose dilation of veins develops the


trophic ulcer of the leg. The bacteriological examination of the ulcer
discharge has revealed Staphylococcus infection. Determine the
antiseptic in the form of ointment from the group of detergents for
local treatment of the ulcer:
A. Aethonium
B. Brilliant green
C. Furacillinum
D. Potassium permanganate
E. Ethacridinum lactas

472. All antiseptics possess all following properties except:


A. Selective antimicrobic action
B. Versatile antimicrobic action
C. Are not introduced parenterally
D. Highly toxic for human
E. Bactericidal action

473. 70% solution of ethyl alcohol is used for surgeon‘s hands


cleaning before operation. Explain the mechanism of action of the
drug:
A. Protein dehydration of microbes protoplasm
B. Interaction with hydroxilic groups of microbes‘ enzymes
C. Interaction with aminogroups of protoplasm proteins of microbes
D. Blockade of sulfhydryl groups of enzymes
E. Oxidation of organic components of microbe‘s protoplasm

474. Which antiseptic is used for surgical tools cleaning?


A. Furacillinum
B. 70% solution of ethyl alcohol
C. 95% solution of ethyl alcohol
D. Ethonium
E. Resorcinum

475. A doctor used 5% spirituous solution of Iodine for operation


field cleaning. Indicate its mechanism of action:
A. Inhibition of dehydrogenase
B. Interaction with amino groups of microbes proteins
that promotes to their denaturation
C. Dehydration of protoplasm‘s proteins
D. Binding to enzymes‘ sulfhydric groups
E. Formation of albuminates

476. Choose the drug for a 6 month old child with mycoplasma
infection:
A. Tetracycline
B. Streptomycin
C. Clotrimazole
D. Erythromycin
E. Penicillin

477. Which antibiotic is not used in myasthenia gravis?


A. Tetracycline
B. Gentamycine
C. Clarithromycin
D. Ceftriaxon
E. Azithromycin

478. A 20 y.o.patient was hospitalized to the surgical department


because of infection caused by blue pus bacillus (Pseudomonas
aeruginosa), which is sensitive to penicillin antibiotics.
Indicate which of the given penicillins has a marked activity on the
Pseudomonas aeruginosa?
A. Benzylpenicillin
B. Methicillin
C. Phenoxymethylpenicillin
D. Carbenicillin disodium
E. Oxacillin

479. The patient with pneumonia was treated with antibiotics for a
long period. After the treatment the patient complains of frequent and
watery stools, abdominal pain. What is the reason of intestinal
disorder?
A. Hereditary enzyme defect
B. Bacteria toxins influence
C. Intestinal dysbacteriosis development
D. Antibiotics toxic influence on the GIT
E. Allergic reaction

480. Purulent endometritis developed in a woman after delivery.


Treating with antibiotic inhibitors of murein synthesis was
ineffective. Wide spectrum bactericidal antibiotic was administered to
her.
In 6 hours t0 rapidly increased up to 400C with shiver. Muscle pains
have appeared. BP dropped down to 70/40 mm Hg. Oliguria has developed.
What is the reason for the development of this condition?
A. Toxic effect of preparation
B. Endotoxic shock
C. Anaphylactic shock
D. Bacteremia
E. Internal bleeding.

481. Choose the drug of choice for cholera prophylaxis:


A. Chloramphenicol
B. Procaine Penicilline
C. Doxycycline
D. Erythromycin
E. All above

482. All of the following clinical indication may require a


combination of antibiotics (rather than a single agent) except:
A. Treatment of mixed infections
B. Treatment of gonorrhea
C. Treatment of tuberculosis
D. Treatment of meningitis
E. Treatment of bacterial endocarditis

483. A 50-year old woman who had underwent chemotherapy for lymphoma a
week ago was brought to the emergency room. She had a fever of 40.3oC
and was confused. Respiration was rapid and BP was 75/40. She was
neutropenic. Gram‘s strains of the urine and sputum are negative.
Which of the following actions is the most beneficial to the patient?
A Send a clinical sample to the laboratory for identification and then
administer an appropriate antibiotic.
B. Administer a combination such as Clindamycin and an
aminoglycoside
C. Administer a broad-spectrum antibiotic like Tetracycline
D. Administer Clindamycin
E. Administer Tobramycinum
484. A 60-year-old alcoholic male with poor dental hygiene is to have
his remaining teeth extracted for subsequent dentures. He has mitral
valve stenosis with mild cardiac insufficiency and is being treated
with captopril, digoxin and furosemide. The dentist decides that his
medical history warrants prophylactic antibiotic therapy prior to the
procedure and prescribes:
A. Vancomycin
B. Amoxicillin
C. Tetracycline
D. Co-trimoxazole
E. Imipenem

485. A 35-year-old man has an infection with Legionella. Assuming no


contraindications, which of the following drugs should you choose?
A. Penicillin
B. Gentamycin
C. Erythromycin
D. Chloramphenicol
E. Tetracycline

486. Which of the following drugs is the most effective agent in the
treatment of Rickettsia,Mycoplasma,and Chlamydia infections?
A. Penicillin
B. Gentamycin
C. Erythromycin
D. Chloramphenicol
E. Tetracycline

487. Streptomycin and other aminoglycosides inhibit bacterial protein


synthesis by binding with one of the following substances.
A. 30S ribosomal particles
B. DNA
C. mRNA
D. Peptidoglycan units in the cell wall
E. RNA polymerase

488. A child who previously was healthy develops bacterial meningitis.


Assuming no specific contraindications, which of the following drugs
will you prescribe?
A. Erythromycin
B. Tetracycline
C. Doxicycline
D. Ceftriaxone
E. Procaine penicilline
489. A patient being treated for springtime allergies with Loratidine
develops an upper respiratory problem. He receives an antibiotic and
develops a cardiac arrhythmia. What kind of antibiotic do you think it
was?
A. Cephaclor
B. Oxacillin
C. Cephalotin
D. Erythromycin
E. Amoxicillin

490. Which of the following drugs is both penicillinase-resistant and


effective by oral administration?
A. Methicillin
B. Carbenicillin
C. Ceftriaxone
D. Amoxicillin plus clavulanic acid
E. Procaine penicillin

491. The penicillin that is effective against Proteus and Pseudomonas


aeruginosa is:
A. Ampicillin
B. Amoxiclav
C. Amoxycillin
D. Carbenicillin
E. Oxacilline

492. Chronic ulceration would most likely occur after administration


of:
A. Carbenicillin
B. Clindamycin
C. Choramphenicol
D. Gentamycin
E. Doxycyclinum

493. Most serious adverse effect of penicillins is:


A. Skin rashes
B. Jarish hexheimer reaction
C. Anaphylaxis
D. Convulsions
E. Urticaria

494. The diagnosis of sepsis has been made to a patient. It was


decided to use a drug from the group fluoroquinolones. Determine this
drug.
A. Erythromycin
B. Ciprofloxacin
C. Gentamycin
D. Tetracyclines
E. Amikacin

495. To treat an exacerbation of chronic pyelonephritis the doctor has


prescribed a drug from the group fluoroquinolones. What is the
mechanism of antimicrobial action of fluoroquinolones?
A. Inhibition of DNA-gyrase
B. Inhibition of synthesis of peptidoglycan
C. Inhibition of protein synthesis by binding to 30S ribosomes
D. Inhibition of protein synthesis by binding to 50S ribosomes
E. Increasing of bacterial wall permeability

496. A patient suffers from severe postoperative pseudomonades


infection. Which antibiotic should be administered in this case?
A.Amikacin sulfate
B.Benzylpenicillin
C.Erythromycin
D.Cephazolin
E.Doxycyclin

497. An antibiotic with ability to penetrate to bone tissues and


cumulate there was administered to 25 y.o. patient with osteomyelitis.
After 3 weeks of using it the patient felt much better. Indicate the
drug:
A. Chloramphenicol (Levomycetin)
B. Oxacillin
C. Ampicillin
D. Penicillin
E. Lincomycin

498. Determine the drug that able to penetrate to bone tissue and bone
marrow to treat bone infections:
A. Gentamicin
B. Neomycin
C. Clindamycin
D. Oleandomycin
E. Erythromycin

499. Staphilococcus aureus resistent to methicillin and other beta-


lactam antibiotics has revealed in bacteriological blood test of a 27
y.o. woman with puerperal sepsis. Which drug should be administered in
this case?
A. Vancomycin
B. Tetracyclin
C. Cefazoline
D. Gentamycin
E. Amphotericin

500. A patient with diminished hearing has severe bacillary infection.


Which pharmacological group is contraindicated to the patient?
A. Tetracyclines
B. Macrolides
C. Tetracyclines
D. Aminoglycosides
E. Fluoroquinolones

501. Infectious agent determined by lab tests is known to be sensitive


to third generation cephalosporins. Choose the drug for treatment:
A. Cefazolin
B. Cephalothin
C. Cephalexine
D. Cefoperazone
E. Ceftriaxone

502. A patient on antimicrobial therapy develops the following signs


and symptoms that ultimately are found to be drug-induced:A cough,
dyspnea, and pulmonary infiltrates; neutropenia and bleeding
tendencies.Which of the following can most likely cause this patients
symptoms?
A. Amoxicillin
B.Ciprofloxacin
C. Azithromycin
D.Furadoninum(Nitrofurantoin)
E. Isoniazid

503. Which of the following drugs is primarily used in amebiasis and


leishmaniasis as well as anaerobic bacterial infections?
A.Co-Trimoxazole (Biseptol)
B. Ciprofloxacin
C. Azithromycin
D. Metronidazole
E. Carbenicillin

504. A patient requires an antibiotic that is the most effective


against P.aeruginosa. Which of the following drugs is the quinolone of
choice?
A. Ofloxacin
B. Ciprofloxacin
C. Lomefloxacin
D. Norfloxacin
E. Enoxacin

505. Which property or mechanism of action is shared by the


penicillins, the cephalosporins, and Amphotericin B?
A. Act through various mechanisms on cell walls or
membranes of susceptible organisms
B. Contraindicated in immunocompromised patients
C. Interact with many drugs by inducing their hepatic metabolism
D. Interact with many drugs by inhibiting their hepatic metabolism
E. Nephrotoxicity precludes use in patients with impaired renal
function

506. A 37 y.o. woman complains of itching in the vulval area. Hanging-


drop examination of the urine reveals trichomonads. Which of the
following drugs is preferred for the treatment of trichomoniasis?
A. Amoxicillin
B. Ciprofloxacin
C. Azithromycin
D. Furadoninum (Nitrofurantoin)
E. Metronidazole

507. The drug of choice for chloromphenicol resistant typhoid fever


is:
A. Cefaclor
B. Cefotaxime
C. Ciprofloxacin
D. Cefazoline
E. Ampicilline

508. A patient has been diagnosed gonorrhea after bacterioscopy of the


smear from urethra. Taking into account that agents of choice for the
treatment of gonorrhea are fluoroquinolones the patient is necessary
to administer:
A. Erythromycin
B. Ciprofloxacin
C. Furasolidone
D. Claritromicin
E. Rifampicin
509. A patient with pneumonia has intolerance of antibiotics. Which
combined sulfonamide preparation should be administered for the
treatment of this patient?
A. Sulfadimidine
B. Sulfadiazine
C. Co-trimoxazole
D. Sulfasalasine
E. Sulfamethoxazole

510. Maximum endocrinal side (gynecomastia, impotence) effects are


seen with:
A. Ketoconazole
B. Fluconazole
C. Myconazole
D. Nystatin
E. Ampicilline

511. Primary mechanism of action of Zidovudine is:


A. Proliferation of B and T cells
B. Inhibition of reverse transcriptase
C. Activation of macrophages
D. Stimulation of lymphocytes
E. Blockade of leukotrien receptors

512. A 35-year-old man under the treatment for pulmonary tuberculosis


has acute-onset of right big toe pain, swelling, and low-grade fever.
The gouty arthritis was diagnosed and high serum uric acid level was
found. Which of the following antituberculosis drugs is known for
causing high uric acid level?
A. Rifampicin
B. Aminosalicylic acid
C. Thiacetazone
D. Pirazinamide
E. Cycloserine

513. A patient with tuberculosis is being treated with Isoniazid. He


develops paresthesias, muscle pain, and unsteadiness. Which vitamin
needs to be given in supplemental doses in order to reverse these
symptoms – or used from the outset to prevent them in high-risk
patients?
A. Vitamin A
B. Vitamin C
C. Vitamin K
D. Vitamin B1 (thiamine)
E. Vitamin B6 (pyridoxine)
514. A 59 y.o. man is diagnosed tuberculosis. Before prescribing a
drug regimen, you take a careful medication history because one of the
drugs commonly used to treat tuberculosis induces microsomal
cytochrome P-450 enzymes in the liver. Which drug is it?
A. Rifampin
B. Vitamin B6 (pyridoxine)
C. Isoniazid
D. Pirazinamide
E. Ethambutol

515. A patient with active tuberculosis is being treated with


Isoniazid and Ethambutol as part of the overall regimen. Which of the
following statements is the main reason for including the Ethambutol?
A. To facilitate entry of Isoniazid into the mycobacteria
B. To facilitate penetration of the blood-brain barrier
C. To slow renal excretion of Isoniazid
D. To retard the development organism resistance
E. To retard absorption after IM injection

516. As a part of a multidrug attack on a patient‘s infection with


Mycobacterium tuberculosis, a physician plans to use an aminoglycoside
antibiotic. Which of the following drugs is the most active against
the tubercle bacillus and seems to be associated with the fewest
problems with resistance or typical aminoglycoside-induced adverse
effects?
A.Amikacin
B. Kanamycin
C. Neomycin
D. Streptomycin
E. Tobramycin

517. Amantadine (Midantanum), used prophylactically against influenza


A2, is thought to act by:
A. Preventing production of viral capsid protein
B. Preventing uncoating of viral DNA
C. Preventing virion release
D. Preventing penetration of the virus into the host cell
E. Causing lysis of infected host cell by release of intracellular
lysosomal enzymes

518. A 30 y.o. man is HIV-positive with a cluster-of-differentiation-


4 (CD4) count of 200/mm3. Within 2 months he develops a peripheral
white blood cell count of 1000/mm3 and a hemoglobin of 9.0 mg/dL.
Which of the following drugs most likely caused the hematological
abnormalities?
A. Rimantadine
B. Midantanum
C. Acyclovir
D. Foscarnet
E. Zidovudine

519. A patient with AIDS is treated with a combination of agents, one


of which is Zidovudine. This drug exerts its main effects by
inhibiting:
A. Viral proteases
B. Viral particle assembly
C. RNA synthesis
D. Nonnucleoside reverse transcriptase
E. Nucleoside reverse transcriptase

520. Which antituberculous agent acts as the competitive antagonist of


para-aminobenzoic acid?
A. Isoniasid
B. Ethambutol
C. Sodium Paraaminosalicilas (PAS)
D. Rifampicin
E. Kanamycin

521. Hyperuricemia is due to inhibition of uric acid secretion in


kidney: gout can occur as a result of adverse effect of:
Ethosuximide
Cycloserine
Pyrazinamide
Rifampicin
Ampicillin

522. A patient was diagnosed with active focal pulmonary tuberculosis.


What drug should be prescribed in the first place?
A. Isoniazid
B. Cyclocerine
C. Ethionamide
D. Ethoxide
E. Sulfalen

523. Pyrantel pamoate is effective in both:


A. Amoebiasis and trichuriasis
B. Taenia solium and ascariasis
C. Amoebiasis and strongyloides
D. Enterobius and ascariasis
E. Strongyloides and enterobius
524. Chloroquine in malaria acts on:
A. Erythrocytic cycle
B. Exoerythrocytic cycle
C. Liver
D. Schizonts
E. All above

525. Flushing occurs after alcohol ingestion in patients taking:


A. Chingamine
B. Penicillin
C. Tetracycline
D. Chloramphenicol
E. Metronidazole

526. Which of the following antimalarial drugs is relatively safe in


pregnancy:
A. Primaquine
B. Co-trimoxazole
C. Chloridine
D. Chingamine
E. Mefloquine

527. Megaloblastic anaemia is a consequence of all drugs except:


A. Trimethoprim
B. Methotrexate
C. Amoxycillin
D. Pyrimethamine
E. Co-Trimoxasole

528. Intraluminal amoebicide of choice is


A. Metronidazole
B. Diloxanide furoate
C. Chloroquine
D. Tetracycline
E. Penicillin

529. To a patient with ascariasis it was given a drug that has also
action on immune system and is used as immunomodulator. What is the
name of this drug?
A. Pyrantel
B. Piperazine adipinate
C. Naphthammone (Bephenium)
D. Levamisole
E. Phenasalum (Niclosamide)
530. A 58-year-old woman has just returned from a trip to Southeast
Asia. Over the past 24 hours she developed shaking, chills, and to of
40.5oC. A blood smear reveals Plasmodium vivax. Which of the following
agents should be used to eradicate the extraerythrocytic phase of the
organism?
A. Chloroquine
B. Primaquine
C. Pyrimethamine
D. Tetracycline
E. Quinacrine (Acrichinum)

531. A young boy presents the infestation with Taenia saginata


(tapeworm). Which of the following drugs is the most appropriate drug
to administer?
A. Mebendazole
B. Niclosamide (Phenasalum)
C. Chloroquine
D. Tetracycline
E. Penicillin

532. In a patient it was revealed mixed helmintic invasion: intestinal


cestodes and liver trematodes. What agent from anthelmintic drugs
should be presvribe?
A. Albendazole
B. Praziquantel
C. Levamisole
D.Pyrantel
E. Piperazine

533. Mother addressed to the pediatrician with the child who


complainted of strong itches in the region around the anus, pain
intensified at night. After investigation of feaces enterobiasis has
been diagnosed. Indicate the drug, which should be administered.
A Piperazine adipinate
B. Trichlorophenum
C. Phenasalum
D. Ditrazinum
E. Aminoacrichinum

534. Indicate the antimalarial agent, which is active against


paraerythrocytic forms of Plasmodium.
A. Amodiachinum
B. Chingaminum
C. Galochinum
D. Hydroxychlorochinum
E. Primachinum

535. For malaria prevention before journey abroad a doctor has got an
agent with histoschizontocidic action. What drug did the doctor take?
A. Primaquine
B. Quinine
C. Doxycyclin
D. Biseptol (Co-trimaxazole)
E. Chloridine (Pyrimethamine)

536. Specify the drug, which is used in amebiasis of any localization


of pathological process.
A Metronidazolum (Trichopolum)
B. Chingaminum
C. Emetinum
D. Chiniophonum
E. Tetracyclinum

537. A woman addressed to a gynecologist in relation with large


discharge from the vagina with unpleasant smell. After laboratory
examination the diagnosis of trichomoniasis in this situation?
A Sulfadimezinum
B. Metronidazolum (Trichopolum)
C. Chingaminum
D. Chloridinum
E. Monomycinum

538. A patient visited a physician with complaints of bowel


dysfunction. After laboratory examination the diagnosis of lambliasis
was made. Specify the drug that should be used:
A Tetracyclinum
B. Trichomonacid
C. Metronidazolum (Trichopolum)
D. Monomycinum
E. Chingaminum

539. During summer vacations a student from tropical country developed


tertian malaria. After recovery he turned back to Ukraine for study
extension. In January an exacerbation was developed. It is known from
past history of disease that drug acting on paraerythrocytic forms of
plasmodium malariae was prescribed. Indicate the drug:
A Chingaminum
B. Halochin
C. Hydroxycholoquine
D. Amodiaquine
E. Primaquine

540. Indicate the alkaloid, which is used for treatment of malaria,


and also possesses other pharmacological activities: decreases
excitabity of the myocardium, stimulates rhythmical contractions of
the uterus, side effects are noise and ringing in the ears, decrease
of hearing:
A Chininin hydrocholoridum
B. Primachine
C. Chloridinum
D. Chingaminum
E. Metronidazolum

541. Indicate the drug that exerts paralyzing action on nematodes,


increases tonicity and contractions of the smooth muscles of the
intestine, so it is used without a purgative agent. It has low
toxicity and is used mainly for ascariasis and entrobiasis:
A. Pyrantale pamoate
B. Mebendazole
C. Primachine
D. Metronidazolum
E. Praziquantel

542. A patient consulted a doctor about bowel disfunction. The doctor


established symptoms of duodenitis and enteritis. Laboratory
examination helped to make the following diagnosis: lambliosis. What
medication should be administered?
A. Monomycin
B. Metronidazole
C. Chingamin
D. Tetracycline
E. Erythromycin

543. For prevention remote relapses of 4-days malaria a 42 y.o.


patient was given primaquine. On the 3rd day of the treatment with
therapeutic doses of the drug patient experienced abdominal pain,
cardiac pain, dyspepsia, generalized cyanosis. What is the reason of
these adverse effects of the drug?
A Cumulation of the therapeutic agent
B. Slowing down of drug‘s eacretion with urine
C. Decreasing of activity of liver microsomal enzymes
D. Genetic deficiency of Glucose-6-phosphate
dehydrogenase
E. Potentiation of the drug‘s action by other therapeutic agent
544. Methotrexate (structural analogue of the folic acid which is
competitive inhibitor of the dihydrofolatreductase) is prescribed for
the treatment of the malignant tumor. On which level does Methotrexate
hinder synthesis of the nucleic acids?
A. Replication
B. Reparation
C. Processing
D. Transcription
E. Mononucleotide synthesis

545. Which phase of the cell cycle is resistant to the most


chemotherapeutic agents, i.e. those that are classified as phase-
specific?
A. Go
B. G1
C. G2
D. M
E. S

546. A cancer man receives prophylactic Allopurinol before a course of


chemotherapy. Which of the following statements is the main purpose of
doing this?
A. Facilitate host cell detoxification of the chemotherapeutic drug,
thereby reducing host cell toxicities
B. Reduce the risk of hyperuricemia and its main
consequences (renal damage, gout) that can occur with a
massive cell kill
C. Inhibit the potential for DNA repair, that otherwise might lead to
chemotherapy failure
D. Potentiate the action of a nitrosourea to bind to purine moieties
in DNA strands
E. Prevent myelosupression and related blood dyscrasias

547. Which of the following statements is the main mechanism by which


the Vincristine exerts its main effects?
A. Alkylating DNA, causing cross-links between parallel DNA strands
B. Blocking microtubular assembly and mitosis during M-
phase
C. Inhibiting topoisomerase, preventing repair of DNA strand breaks
D. Intercalating in DNA strands, thereby preventing DNA replication by
mRNA
E. Stabilizing assembled microtubular arrays, thereby preventing
mitosis
548. Which of the following is the main mechanism by which the
Cyclophosphamide exerts its cell killing?
A. Alkylating DNA, causing cross-links between parallel
DNA strands
B. Blocking microtubular assembly and mitosis during M-phase
C. Inhibiting topoisomerase, preventing repair of DNA strand breaks
D. Intercalating in DNA strands, thereby preventing DNA replication by
mRNA
E. Stabilizing assembled microtubular arrays, thereby preventing
mitosis

549. Which of the following conditions is the most likely adverse


response to occur as a result of the Vincristine action?
A. Nephrotoxicity, renal dysfunction or failure
B. Peripheral sensory and motor neuropathy
C. Pulmonary damage
D. Agranulocytosis
E. Rhabdomyolysis

550. A patient with advanced Hodgkin‘s disease is placed on


combination therapy with Vincristine, Embichinum (Chlormethine),
Procarbazine, and Prednisone (the so-called MOPP regimen). Which of
the following procedures plays the main role of the Prednisone effects
in this therapeutic plan?
A. Preventing opportunistic infections
B. Exerting direct cytotoxic actions, independent of the other drugs
C. Counteracting fluid overload from chemotherapy-induced renal
dysfunction
D. Counteracting hyperglycemia caused by the other agents
E. Suppressing emesis and vomiting

551. A 25-year-old-woman with choriocarcinoma is treated with very


high doses of Methotrexate. You anticipate significant host cell
toxicity in response to the high Methotrexate dose, and so immediately
after giving the anticancer drug you administer one of the following
drugs:
A. Vitamin K
B. Vitamin B1
C. Vitamin B6
D. Vitamin B12
E. Folic acid

552. While reviewing charts in a general medicine clinic you see that
a woman, 27 y.o. and with no history of cancer at all, is also taking
Methotrexate. The drug is most likely given to manage one of the
following conditions. What is it?
A. Hyperthyroidism
B. Asthma or emphysema
C. Clinical gout
D. Myasthenia gravis
E. Rheumatoid arthritis or psoriasis

553. Allopurinol should be avoided, or reduced doses of the


chemotherapeutic agent, if the anticancer drug is one of the
following:
A. Doxorubicin
B. Cysplatin
C. Mercaptopurine
D. Cyclophosphamide
E. Vincristine

554. A patient with Wilm‘s tumor is receiving a chemotherapeutic agent


that is described as acting by intercalating into DNA strands, and
that is efficacious regardless of what stage of the cell cycle the
tumor cell are in. Which of the following agents best fits this
description?
A. Dactinomycin (Actinomycin)
B. Cytarabin (Cytosine arabiniside)
C. Mercaptopurine
D. Cyclophosphamide
E. Vincristine

555. A cancer patient develops severe, irreversible cardiomyopathy


because the maximum dose of an anticancer drug was exceeded. Which of
the following drugs is most likely responsible for this patient‘s
symptoms?
A. Doxorubicin
B. Cysplatin
C. Mercaptopurine
D. Cyclophosphamide
E. Vincristine

556. A 45 y.o. man complains of progressive difficulty starting his


stream urinating, and having to get up at least once night to urinate.
Rectal examination reveals an enlarged, smooth-surfaced prostate.
Prostatic serum antigen (PSA) titers are elevated.Urine flow
increases,and prostate size decreases,in response to Cyproterone
acetate_(Androcur) treatment. This drugs main mechanism of action
involves one of the following processes.What is it?
A.5±-reductase \inhibition
b.±1-adrenergic receptor blockade
C. Lowering serum Testosterone levels
D. Testosterone synthesis inhibition
E. Competition with dihydrotestosterone for
intracellular androgen receptor and inhibition its
binding

557. The drug used to treat Methotrexate toxicity is:


A. Folic acid
B. Folinic acid
C. Riboflavin
D. Cyanocobalamin
E. Vicasol

558. The following drugs are alkylating agents except:


A. Cyclophosphamide
B. Methotrexate
C. Chlorambucil
D. Sarcolysinum
E. Myelosanum

559. The antitumoral agent from the group of antimetabolites


(antagonist of the folic acid) was administered to the patient with
acute leucosis. Indicate this drug.
A. Methotrexatum
B. Fluorouracil
C. Myelosanum
D. Mercaptopurine
E.Hexestrol

560. Determine the antitumoral drug from the group of antimetabolites


which is used for treatment of leucosis in children and cancer in
adults:
A Methotrexate
B. Sarcolysinum
C. Colchamine
D. Rubomycin
E.Predrnisolone

561. A patient takes an acute, massive overdose of aspirin that,


without proper intervention, will be fatal. Which of the following
conditions would you expect in the advanced (late) stages of aspirin
(salicylate poisoning)?
A. Metabolic alkalosis
B. Respiratory acidosis
C. Respiratory alkalosis, then - metabolic acidosis
D. Hypothermia
E. Ventilatory stimulation

562. In addition to providing symptomatic, supportive care, which of


the following drugs would be a helpful adjunct to manage severe
Paracetamol (Acetaminophen) poisoning?
A. Naloxon
B. Acetylscystein
C. Diazepam
D. Sodium bicarbonate
E. Unitiolum

563. Which of the following conditions is the primary cause of death


from massive Acetaminophen overdoses?
A. Acute nephropathy
B. Status epilepticus
C. Status asthmaticus
D. A-V conduction disturbances
E. Liver failure

564. A patient has taken a potentially lethal dose of Acetaminophen


(Paracetamol). The current preferred antidotal therapy involves
administration of drug that:
A. Inhibits synthesis of superoxide anion radical and hydrogen
peroxide
B. Is rich in sulfhydryl (-SH) groups
C. Alkalinizes the urine to facilitate Acetaminophen excretion
D. Inhibits hepatic oxidative metabolism to inhibit formation of
Acetaminophen‘s toxic metabolites
E. Causes metabolic acidosis to combat the toxic metabolite‘s
metabolic alkalosis

565. A patient who receives a rapid IV injection of a drug develops


hypocalcemic tetany. Which of the following drugs is the most likely
cause it?
A. Edetate Sodium (Trilon B)
B. Unitiolum
C. Penicillamine
D. Deferoxamine
E. Acetylscysteine

S-ar putea să vă placă și